Final Exam Practice Questions

अब Quizwiz के साथ अपने होमवर्क और परीक्षाओं को एस करें!

A "review of systems" is ALWAYS: A) subjective: part of the history B) objective: part of the physical exam C) part of the assessment section D) part of the plan section

A) subjective: part of the history The ROS is ALWAYS subjective.

The components of the health history include all of the following except which one? A. Review of systems B. Thorax and lungs C. Present illness D. Personal and social items

B. Thorax and lungs The thorax and lungs are part of the physical examination, not part of the health history. The other answers are all part of a complete health history.

Rhonda is seeing a patient that just came back from the beach for spring break and has a rash on her thighs that started after the last day while she was laying in the sun. A cursory exam of her thigh and the rash looks like a hand print. All of the following are important questions to ask, EXCEPT: A) Did you wear sunscreen? B) Did you shower after being in the sun? C) What were you drinking while you were laying out? D) Did you see any sand fleas?

D) Did you see any sand fleas?

A man's wife is upset because when she hugs him with her hands on his left shoulder blade, "it feels creepy." This came on gradually after a recent severe left-sided rotator cuff tear. How long does it usually take to develop muscular atrophy with increased prominence of the scapular spine following a rotator cuff tear? A) 1 week B) 2-3 weeks C) 1 month D) 2-3 months

B) 2-3 weeks Prominence of the scapular spine occurs with generalized muscle wasting as well as with specific injuries such as a rotator cuff tear. It is easily palpable, even through indoor clothing, although the back should be exposed to make other important observations. Atrophy usually occurs several weeks following a rotator cuff tear

A young woman undergoes cranial nerve testing. On touching the soft palate, her uvula deviates to the left. Which of the following is likely? A) CN IX lesion on the left B) CN IX lesion on the right C) CN X lesion on the left D) CN X lesion on the right

D) CN X lesion on the right

Is the following statement true or false? When assessing the patient's thought content, it is important to always follow specific questions to keep the patient on task.

False. When assessing the patient's thought content, follow appropriate leads as they occur rather than using stereotyped lists of specific questions.

Which is palpable deep to the left costal margin during inspiration? a) liver edge b) spleen edge

b) spleen edge

Which of the following is true about hallucinations? a.Experiences may or may not be recognized by the person as false b.Hallucinations may be auditory, visual, olfactory, gustatory, tactile, or somatic c.Do not include false perceptions associated with dreaming and falling asleep d.All of the above

d. All of the above -Experiences may or may not be recognized by the person as false -Hallucinations may be auditory, visual, olfactory, gustatory, tactile, or somatic -Do not include false perceptions associated with dreaming and falling asleep

A 32-year-old attorney comes to your clinic complaining of severe neck pain when he tries to turn his head to the right. He reports to you that he was the restrained driver in a MVA the day before, when he was hit from behind at approximately 40 mph. He states he did not hit his head and he did not lose consciousness. He refused to be evaluated by EMS at the scene. He states he felt okay after the accident but today woke up in extreme pain. He is having no numbness or tingling in his right hand and states he has no problems holding a pen. He has no symptoms below the neck. He has no significant past medical history. On exam, he has difficulty turning his head to the right or tilting it to the right. He also has difficulty with extending his head. He is able to flex his neck and turn and tilt to the left. On palpation, he is tender over the right trapezius and rhomboid muscles. He has no tenderness along the cervical vertebrae or the clavicle or scapula. He has normal reflexes in his biceps, triceps, and brachioradialis. He has normal strength in his elbow and wrist and has normal two-point discrimination. Having the patient cough does not increase his symptoms. What type of neck pain best describes his problem? (A) Cervical sprain (B) Muscle ache (C) Neck pain with dermatomal radiation (D) Neck pain from cervical spinal stenosis

(A) Cervical sprain A cervical sprain is often associated with an injury such as occurs in a motor vehicle accident, heavy lifting, or impact against the neck. There is local tenderness over the cervical muscles and a decreased range of motion of the neck.

A 78-year-old grandmother presents for evaluation of weakness in her face. She has a long-standing history of hypertension that has been under fair control. On physical examination, you note that she has ptosis and miosis of the left eye, and left facial anhidrosis. What is your most likely diagnosis? (A) Horner's syndrome (B) Anisocoria (C) Acute angle-closure glaucoma (D) Myasthenia gravis

Horner's syndrome

The primary muscles of respiration include the: (A) Diaphragm and intercostals (B) Trapezius and rectus abdominus (C) Sternomastoids and scalenes (D) External obliques and pectoralis major

(A) Diaphragm and intercostals The major muscle of respiration is the diaphragm. The intercostals muscles lift the sternum and elevate the ribs during inspiration, increasing the anteroposterior diameter.

A 22-year-old agricultural worker presents to your office for evaluation of coughing. She has had this cough for 6 weeks; it is nonproductive and worst first thing in the morning. She denies fever, chills, weight loss, or night sweats; she does have clear rhinorrhea and itchy, watery eyes. You diagnose her with allergic rhinoconjunctivitis. On physical examination of the nasal mucosa, what would you expect to find? (A) Erythematous, ulcerated mucosa (B) Pale to bluish, boggy mucosa (C) Pink mucosa (D) Ulcerated mucosa

(B) Pale to bluish, boggy mucosa Allergic rhinoconjunctivitis is a common chronic disorder in children, especially in developed countries. It does not cause nasal symptoms only (such as congestion and sneezing) but may also cause general complaints such as fatigue and cough.

Next, you place a vibrating tuning fork on the teenager's right mastoid process, asking him to let you know when the sound is gone, then immediately place that same tuning fork near his right ear. He hears the sound equally in air as against his bone. The name of this test is: (A) Weber's test (B) Rinne's test (C) Whisper test (D) Romberg test

(B) Rinne's test

The most important technique when progressing from one auscultory site on the thorax to another is: (A) Top-to-bottom comparison (B) Side-to-side comparison (C) Posterior-to-anterior comparison (D) Interspace-by-interspace comparison

(B) Side-to-side comparison Side to side comparison, the "ladder" technique, helps pinpoint any asymmetries in the quality of the breath sounds. Listen to at least one full respiration in each location.

While auscultating the lungs of an obese patient, you would expect the heart sounds to be: (A) Louder and closer (B) Softer and more distant (C) Louder and more distant (D) Softer and closer

(B) Softer and more distant Fat and muscle can muffle heart sounds, making them more distant.

When auscultating the chest in an adult, you would: (A) Use the bell of the stethoscope held lightly against the chest to avoid friction (B) Use the diaphragm of the stethoscope held firmly against the chest (C) Instruct the client to breathe in and out through her nose (D) Instruct the patient to take deep, rapid breaths

(B) Use the diaphragm of the stethoscope held firmly against the chest The diaphragm of the stethoscope held firmly on the chest is the correct way to auscultate breath sounds. The patient should be instructed to open the mouth and breathe in and out more deeply than usual, but not to hyperventilate.

Air passing through narrowed bronchioles would produce which of the following adventitious sounds: (A) Whispered pectoriloquy (B) Wheezes (C) Bronchophony (D) Muffled breath sounds

(B) Wheezes Wheezes are high-pitched musical breath sounds caused by air squeezed or compressed through narrowed inflamed small- and middle-caliber airways. Whispered pectoriloquy occurs when whispered voice sounds are transmitted more clearly through collapsed alveoli. Bronchophony results from increased transmission of spoken voice sounds. Usually muffled or absent breath sounds result from collapse, not just narrowing, of the bronchioles.

A 2-week-old infant is brought into your clinic by his parents because they have noticed that their child is having some difficulty with feeding and that there is something strange in his mouth. You diagnose thrush. What is the most likely physical finding you will see upon examination of his mouth? (A) Koplik's spots (B) White plaques (C) Erythematous plaques (D) Epstein's pearls

(B) White plaques Koplik's spots (also Koplik's sign) are a prodromic viral enanthem of measles manifesting two to three days before the measles rash itself. They are characterized as clustered, white lesions on the buccal mucosa. Epstein Pearls are very small cysts that can appear in a baby's mouth that look like tiny, white bumps. They generally appear along a baby's gums or along the top of the roof of the mouth. They are seen in 60% to 85% of newborns.

Tachypnea, use of accessory muscles, prolonged expiration, intercostal retraction, decreased breath sounds, and expiratory wheezes are all symptomatic of: (A) Pleural effusion (B) Atelectasis (C) Asthma (D) Bronchitis

(C) Asthma Asthma arises from hypersensitivity to allergenic inhaled particles, producing inflammation and smooth muscle constriction in the bronchioles and increased airway resistance during expiration. This produces increased work of breathing and use of accessory muscles.

A 19-year-old college student presents to your office for evaluation of episodes of chest pain and shortness of breath. She has no medical problems. She does not smoke, drink alcohol, or use illicit drugs. She is on the dean's list and is studying electrical engineering. On auscultation of her heart, you hear a mid-systolic click at the apex. What is her most likely diagnosis? (A) Bicuspid aortic valve (B) Pulmonary hypertension (C) Mitral valve prolapse (D) Mitral stenosis

(C) Mitral valve prolapse Given the patient scenario and the location and timing of the sound, the most likely diagnosis is mitral valve prolapse. The click is heard best at or medial to the apex.

Expected findings in the healthy adult lung include the presence of: ( A) Increased tactile fremitus and dull percussion tones (B) Adventitious sounds and limited chest expansion (C) Muffled voice sounds and symmetrical tactile fremitus (D) Absent voice sounds and hyperresonant percussion tones

(C) Muffled voice sounds and symmetrical tactile fremitus Normal lung findings include: symmetric chest expansion, resonance on percussion, vesicular breath sounds over the lung fields, no adventitious sounds, and muffled voice sounds.

You are examining a child with severe cerebral palsy. When you suddenly move his foot dorsally, a sustained "beating" of the foot against your hand ensues. What does this represent? A) focal seizure B) clonus C) extinction D) reinforcement

B) clonus

A 57-year-old auto mechanic comes to your office for evaluation of decreased vision in his right eye. You perform a fundoscopic examination and diagnose a cataract. What did you see to make this diagnosis? (A) Absence of the red reflex (B) Neovascularization (C) Hemorrhage in the fundus (D) Uveitis

(A) Absence of the red reflex

A 42-year-old receptionist presents to the office for evaluation of fatigue. The fatigue has been present for several months. Upon auscultation of the heart, the S1 sound is accentuated. Which is the most likely cause of her accentuated S1 given her symptoms? (A) Anemia (B) Mitral regurgitation (C) Right bundle branch block (D) Atrial fibrillation

(A) Anemia An accentuated S1 occurs in tachycardia, rhythms with a short PR interval, and in high cardiac output states. Anemia is one cause of a high cardiac output state and, along with this patient's fatigue, is the most likely diagnosis.

A patient demonstrates left facial droop but can wrinkle both sides of the forehead. The examiner should suspect a problem with: (A) Central innervation of cranial nerve VII (facial) (B) Cranial nerve V (trigeminal) (C) Peripheral innervation of cranial nerve VII (D) Frontalis muscle

(A) Central innervation of cranial nerve VII (facial) A review of the cranial nerves and their function support the symptoms presented. Facial muscles are mediated by cranial nerve Vll.

Dullness on percussion over the left lower lobe of the lung is most likely to reflect: (A) Consolidation (B) Asthma (C) Chronic obstructive pulmonary disease (D) Excess adipose tissue

(A) Consolidation A dull percussion note indicates loss of aerated lung tissue (alveoli), or consolidation, as in pneumonia, pleural effusion, atelectasis, or tumor. In asthma and COPD, there is often hyperresonance from air trapping due to narrowing of the airways on expiration. Adipose tissue is more likely to muffle breath sounds than to change the percussion note over normally aerated lung tissue.

A patient complains of shortness of breath and productive cough. Consolidation is present in the lungs if you find: (A) Dullness to percussion over left base (B) Bronchial breath sounds throughout (C) Increased tactile fremitus throughout (D) Inspiratory and expiratory wheezes

(A) Dullness to percussion over left base When alveoli are filled with purulent material, as in pneumonia, there is consolidation in the alveoli causing dullness to percussion. Breath sounds in that area may be bronchial, or harsh, or even absent. Tactile fremitis is increased, but only in the area of consolidation and not throughout the lungfields. Inspiratory and expiratory wheezes suggest narrowed airways as in asthma.

A 3-year-old boy is brought to your office by his mother for evaluation of fever, loss of appetite, and emesis. The symptoms have been present for 2 days; the fever is only temporarily relieved with antipyretics. You perform a physical examination and diagnose otitis media. What is your most likely physical finding on otoscopic examination? (A) Erythematous, bulging tympanic membrane (B) Erythematous, retracted tympanic membrane (C) Erythematous, scaly external ear canal (D) Erythematous helix

(A) Erythematous, bulging tympanic membrane

A 7-year-old elementary school child is brought into the clinic by her mother for evaluation of fever and nausea. Upon further physical examination you diagnose tonsillitis. What are you typically expecting to see on physical examination of the oropharynx? (A) Exudates on the tonsils (B) Small tonsils (C) Hemorrhage of the tonsils (D) Pseudomembranes

(A) Exudates on the tonsils Tonsillitis is an inflammation of the two oval-shaped pads of tissue at the back of the throat. Symptoms include sore throat, difficulty swallowing, and tender lymph nodes. The tonsil may develop a gray or white coating (exudate).

A 5-year-old child presents with his parents for a regular checkup. He has normal growth and development and is appropriately physically active. On auscultation of his heart, you hear a grade 2 murmur at the 2nd through 4th left interspaces between the left sternal border and the apex. It has a medium pitch, and the quality of the sound is variable. It does not radiate. It disappears when the patient is sitting. What is the most likely cause of this murmur? (A) Innocent (B) Pulmonic stenosis (C) Physiologic (D) Bicuspid aortic valve

(A) Innocent This is the most likely cause. There are no associated findings of any pathologic or cardiovascular disease.

The following findings indicate a possible pulmonary abscess: (A) Malodorous breath (B) Protrusion of the clavicle (C) Clubbing of the nail beds (D) Kussmaul respirations

(A) Malodorous breath Lung abscesses are usually anaerobic, causing an unpleasant odor. The clavicle is not affected by a lung abscess. Clubbing is seen in chronic lung or liver disease and in congenital heart disease. Kussmaul breathing, or rapid deep breathing, is seen in anxiety, exercise, and metabolic acidosis.

A 52-year-old computer analyst presents to the office for a checkup. He is doing well except that he has noticed increasing shortness of breath with physical exertion, so he has cut down on his recreational activities. On auscultation of the heart, you hear a blowing, medium-pitched pansystolic murmur at the apex. It radiates to the left axilla and does not change with inspiration. This murmur is most likely: (A) Mitral regurgitation (B) Tricuspid regurgitation (C) Ventricular septal defect (D) Aortic stenosis

(A) Mitral regurgitation The murmur of mitral regurgitation is best heard at the apex and radiates to the left axilla. It is soft-to-loud in intensity with a medium-to-high pitch and a blowing quality. It does not become louder with inspiration.

When percussing normal lungs, the expected percussion note would be: (A) Resonance (B) Tympany (C) Dullness (D) Stridor

(A) Resonance Resonance is the expected finding in normally aerated lung tissue.

The examiner notes an abnormally high diaphragm on the right side and descent of 4 cm on the left side. These findings suggest: (A) The patient may have a pleural effusion (B) The patient may have right middle lobe pneumonia (C) Asymmetrical findings, which are common in well-conditioned adults (D) A normal finding because the right lung is larger than the left lung

(A) The patient may have a pleural effusion In pleural effusions, fluid accumulates in the pleural space, separating air-filled lung from the chest wall, blocking transmission of sound from underlying lung tissue. This reduces detection of movement of the diaphragm on percussion. A right middle lobe pneumonia would not affect descent of the diaphragm. A pneumothorax is air leakage into the pleural space, causing loss of aerated lung tissue but not necessarily impairing descent of the diaphragm. Normal lung findings are usually symmetrical in healthy adults. The right lung has three lobes compared to two on the left but is not significantly larger.

A 28-year-old housewife presents to your office for a 6-week-postpartum checkup. She complains of fatigue greater than expected and palpitations. Her hair is falling out as well. She denies sadness or depression symptoms. Before this, she had not had any medical problems. She is breast-feeding her child and is not on any birth control. She had her first period since giving birth last week. A pregnancy test done in the office is negative. What is your most likely diagnosis? (A) Thyroiditis (B) Iron-deficiency anemia (C) Addison's disease (D) Sheehan's syndrome

(A) Thyroiditis Thyroiditis is inflammation of the thyroid, the butterfly-shaped gland in the neck.

When you look at the tympanic membrane with the otoscope, what normal landmark is present? (A) Umbo (B) Otoliths (C) Stapes (D) Tragus

(A) Umbo situated at the tip of the manubrium of the malleus, which is fused with the tympanic membrane

A 15-year-old member of the high school marching band comes to your office for evaluation of hearing loss. He had multiple ear infections as an infant and toddler, and had to have myringotomy tubes inserted in his ears. Additionally, he suffers from many allergies. His hearing is diminished in the right ear. When you place a vibrating tuning fork on the top of his head, the sound lateralizes to the right ear. The name of this test is: (A) Weber's test (B) Rinne's test (C) Whisper test (D) Romberg test

(A) Weber's test

Decreased breath sounds would be most likely to occur: (A) When the bronchial tree is obstructed (B) When adventitious sounds are present (C) In conditions of hyperresonance like COPD (D) In conjunction with whispered pectoriloquy

(A) When the bronchial tree is obstructed Decreased or absent breath sounds occur when the bronchial tree is obstructed as in pneumonia or pleural effusion. Adventitious sounds are extra sounds such as rales, wheezes, or rhonchi, heard in congestive heart failure, asthma, and bronchitis, respectively. In COPD, there is often delayed expiration, but breath sounds are still heard. Whispered pectoriloquy pertains to the quality of transmitted spoken words, not breath sounds.

A 50-year-old truck driver presents to your office for a routine physical examination. He denies any medical problems; he has never had surgery; he takes no medications or over-the-counter supplements. He has smoked 1 pack of cigarettes daily for 30 years. He does not use alcohol or illicit drugs. His family history is remarkable for hypertension and stroke. On physical examination, you notice that one pupil is larger than the other by 0.4 mm; everything else is normal. What is the most likely diagnosis? (A) Horner's syndrome (B) Anisocoria (C) Brain tumor (D) Tonic pupil

(B) Anisocoria Anisocoria is a condition characterized by an unequal size of the eyes' pupils. Affecting 20% of the population, it can be an entirely harmless condition or a symptom of more serious medical problems.

A 38-year-old warehouse stocker presents to your office complaining of a headache. He has had these headaches intermittently for several years, but they have been increasing in frequency. In your clinic today, his blood pressure is 170/110 mm Hg. His urine dipstick is positive for proteinuria only, and his fingerstick glucose is 100 mg/dL. You diagnose him with uncontrolled hypertension. What finding on fundoscopic examination would support this diagnosis? (A) Cotton-wool spots (B) Arteriovenous (AV) nicking (C) Blurred optic disc margins (D) Macular star

(B) Arteriovenous (AV) nicking

A 25-year-old veterinarian presents to the clinic for evaluation of flank pain, dysuria, nausea, and fever. A urine pregnancy test is negative. A urine dipstick is positive for leukocyte esterase. On physical examination, what would be the most likely sign expected? (A) Psoas sign (B) CVA tenderness (C) Rovsing's sign (D) Murphy's sign

(B) CVA tenderness

When performing posterior palpation of the thyroid gland, you should do all of the following EXCEPT: (A) Have the patient tip his or her head forward and slightly to the side. (B) Place your index fingers above the cricoid cartilage. (C) Palpate between the sternocleidomastoid muscle and the trachea for the thyroid isthmus. (D) Move your fingers laterally to palpate for the thyroid lobes.

(B) Place your index fingers above the cricoid cartilage. The patient is examined in the seated or standing position. Have the patient tip his or her head forward and slightly to the side. Standing behind the patient, attempt to locate the thyroid isthmus by palpating between the cricoid cartilage and the suprasternal notch. (BELOW the cricoid cartilage) Move your hands laterally to try to feel under the sternocleidomstoids for the fullness of the thyroid. Have the patient swallow a sip of water as you palpate, feeling for the upward movement of the thyroid gland.

A 38-year-old file clerk presents to your office for evaluation of fatigue. She has a 20-pack-ayear history of smoking and had rheumatic heart fever in childhood. On auscultation of her heart, the S2 sound is widely split; that is, it persists throughout the respiratory cycle. What is the most likely cause of the widely split S2? (A) Physiologic cause (B) Pulmonic stenosis (C) Atrial septal defect (D) Left bundle branch block

(B) Pulmonic stenosis Pulmonic stenosis means that the pulmonic valve has delayed closure. This results in a widely split S2, which means that the usual splitting is increased and persists throughout the respiratory cycle (including expiration).

A 62-year-old construction worker presents to your office for evaluation of fatigue. He has an elevated blood pressure with a widened pulse pressure. Upon auscultation of his heart, you hear a mid-systolic murmur at the right second interspace that radiates to the neck. It has a medium pitch and a harsh quality. It is best heard when you sit the patient up and lean him forward. What is your most likely diagnosis? (A) Physiologic (B) Pulmonic stenosis (C) Aortic stenosis (D) Hypertrophic cardiomyopathy

(C) Aortic stenosis In aortic stenosis, the murmur is located in the right 2nd interspace and often radiates into the neck and down the left sternal border, even to the apex. It is often loud and has a medium pitch with a harsh quality. It is best heard with the patient sitting up and leaning forward.

The patient has an undiagnosed tumor in the middle lobe of the right lung, causing atelectasis, as suggested by: (A) Low-pitched grating sound heard during inspiration and expiration (B) Hyperresonance in the right middle lobe (C) Diminished or absent breath sounds in the right middle lobe (D) An ammonia-like odor on the patient's breath

(C) Diminished or absent breath sounds in the right middle lobe Atelectasis occurs when air flow is obstructed by a tumor, a mucous plug, or a foreign object; the alveoli of affected lung tissue collapse into an airless state. Percussion reveals dullness over the airless area, and breath sounds are absent. Odor and hyperresonance are not consistent with atelectasis.

A patient presents with an area of dullness to percussion and breath sounds that are decreased to absent, suggesting the following diagnosis: (A) Pneumothorax (B) COPD (emphysema) (C) Pleural effusion (D) Asthma

(C) Pleural effusion In pneumothorax and COPD, there is hyperresonance and decreased-to-absent breath sounds. In asthma, there is resonance or hyperresonance from air trapping, but breath sounds are often obscured by wheezing.

Which of the following is the best technique for assessing the supraclavicular lymph nodes? (A) Place the patient in a supine position and ask him to hold his breath while you palpate (B) Place the patient in Trendelenburg position and illuminate the nodes with a bright light (C) Standing behind the patient, palpate deeply behind the clavicles as he takes a deep breath (D) Palpate lightly below the clavicles with the patient in a sitting position

(C) Standing behind the patient, palpate deeply behind the clavicles as he takes a deep breath When the patient inspires, the supraclavicular lymph nodes may be more easily palpable, especially from a posterior position.

To rule out a middle lobe pneumonia, you must make sure to auscultate: (A) Beneath the right breast (B) Beneath the left breast (C) Under the right axilla (D) Under the left axilla

(C) Under the right axilla Only the right lung had a middle lobe. You must also examine the remaining areas of both lung fields to compare the quality of breath sounds at several locations.

A teenage boy presents to the emergency room with complaints of sharp pain and trouble breathing. You find that the patient has cyanosis, tachypnea, tracheal deviation to the right, decreased tactile fremitus on the left, hyperresonance on the left, and decreased breath sounds on the left. This is consistent with: (A) Acute pneumonia (B) An asthmatic attack (C) Bronchitis (D) A spontaneous pneumothorax

(D) A spontaneous pneumothorax Young men may present with acute shortness of breath from spontaneous pneumothorax of unknown etiology. With a pneumothorax, free air in the pleural space causes partial or complete lung collapse and deviation of the trachea to the nonaffected side. If the pneumothorax is large, tachypnea and cyanosis may occur.

A 75-year-old retired farmer presents to your office for a hospital follow-up visit. He was diagnosed with a stroke and placed on medication. On review of the hospital chart, he was noted to have a stroke in the optic chiasm. What findings do you expect on examination of his visual fields? (A) Right homonymous hemianopsia (B) Horizontal defect (C) Bilateral visual field obliteration (D) Bitemporal hemianopsia

(D) Bitemporal hemianopsia

A 15-year-old high school student presents to your clinic complaining of pain in his left ear. He noticed that it occurred shortly after starting swimming lessons at the local YMCA. Upon physical examination, you notice that the external canal is swollen and tender during insertion of the speculum. What is your most likely diagnosis? (A) Otitis media (B) Serous otitis (C) Eustachian tube dysfunction (D) Otitis externa

(D) Otitis externa Otitis externa (also known as swimmer's ear) is a condition that causes inflammation (redness and swelling) of the external ear canal, which is the tube between the outer ear and eardrum.

When considering how to phrase your documentation, all of the following are examples of acceptable phrasing EXCEPT: A) Lungs: lungs auscultated B) Neck: trachea midline C) Abdomen: no tenderness or masses D) Skin: nails without clubbing

A) Lungs: lungs auscultated

The assessment tool use to measure sexual maturity in girls is called: A) Tanner Scale B) Developmental Scale C) Breast and Vaginal Scale D) Wilson Scale

A) Tanner Scale

You are assessing a patient for peripheral vascular disease in the arms, secondary to a complaint of increased weakness and a history of coronary artery disease and diabetes. You assess the brachial and radial pulses and note that they are bounding. What does that translate to on a scale of 0 to 3? A) 0 B) 3+ C) 2+ D) 1+

B) 3+ A pulse of 3+ is considered to be bounding.

A mother of a 1-month-old infant asks the nurse when her baby will stop automatically grabbing her finger when she puts it in the baby's hand (palmar grasp). The correct response is: A) 2 months B) 4 months C) 6 months D) 8 months

B) 4 months The palmar grasp disappears around 3-4 months.

When is the best time to tell a female patient to examine her breasts? A) 2 weeks after the first day of her last period (ovulation) B) 5-7 days after the first day of her last period C) during the menstrual week D) day before she is supposed to start her period

B) 5-7 days after the first day of her last period

Jacob, a 33-year-old construction worker, complains of a "lump on his back" over his scapula. It has been there for about a year and is getting larger. He says his wife has been able to squeeze out a cheesy-textured substance on occasion. He worries this may be cancer. When gently pinched from the side, a prominent dimple forms in the middle of the mass. What is most likely? A) An enlarged lymph node B) A sebaceous cyst C) An actinic keratosis D) A malignant lesion

B) A sebaceous cyst This is a classic description of an epidermal inclusion cyst resulting from a blocked sebaceous gland. The fact that any lesion is enlarging is worrisome, but the other descriptors are so distinctive that cancer is highly unlikely. This would be an unusual location for a lymph node, and these do not usually drain to the skin.

Which of the following occurs in respiratory distress? A) speech is effortless B) skin between the ribs moves inward with inspiration due to accessory muscle use C) neck muscles are relaxed D) patient torso leans posteriorly

B) skin between the ribs moves inward with inspiration due to accessory muscle use

A 25-year-old woman in her fifth month of pregnancy has a blood pressure of 100/70 mm Hg. In reviewing her previous exam, the nurse notes that her blood pressure in her second month was 124/80 mm Hg. In evaluatin this change, what does the nurse know to be true? A) because of increased cardiac output, the blood pressure should be higher this time B) this is the result of peripheral vasodilation and is an expected change C) this is not an expected finding because it would mean a decreased cardiac output D) this would mean a decrease in circulating blood volume, which is dangerous for the fetus

B) this is the result of peripheral vasodilation and is an expected change

A 30-year-old woman is visiting the clinic because of "pain in my bottom when I have a bowel movement." The nurse should assess for which problem? A)Pinworms B)Hemorrhoids C)Colon cancer D)Fecal incontinence

B)Hemorrhoids Having painful bowel movements, known as dyschezia, may be due to a local condition (hemorrhoid or fissure) or constipation. The other responses are not correct.

A 54-year-old woman who has just completed menopause is in the clinic today for a yearly physical examination. Which of these statements should the nurse include in patient education? "A postmenopausal woman: A)is not at any greater risk for heart disease than a younger woman is." B)should be aware that she is at increased risk for dyspareunia because of decreased vaginal secretions." C)has only stopped menstruating; there really are no other significant changes with which she should be concerned." D)is likely to have difficulty with sexual pleasure as a result of drastic changes in the female sexual response cycle."

B)should be aware that she is at increased risk for dyspareunia because of decreased vaginal secretions." Decreased vaginal secretions leave the vagina dry and at risk for irritation and pain with intercourse (dyspareunia). The other statements are incorrect.

The lifetime risk for a woman to have breast cancer is: A) 2% B) 5% C) 12% D) 20%

C) 12% The lifetime risk of breast cancer is 12%, or 1 in 8 women.

Which of the following is the most common type of hernia in men? A) femoral B) direct C) indirect D) ventral

C) indirect

A 21-year-old college senior presents to your clinic, complaining of shortness of breath and a nonproductive nocturnal cough. She states she used to feel this way only with extreme exercise, but lately she has felt this way continuously. She denies any other upper respiratory symptoms, chest pain, gastrointestinal symptoms, or urinary tract symptoms. Her past medical history is significant only for seasonal allergies, for which she takes a nasal steroid spray but is otherwise on no other medications. She has had no surgeries. Her mother has allergies and eczema and her father has high blood pressure. She is an only child. She denies smoking and illegal drug use but drinks three to four alcoholic beverages per weekend. She is a junior in finance at a local university and she has recently started a job as a bartender in town. On examination she is in no acute distress and her temperature is 98.6. Her blood pressure is 120/80, her pulse is 80, and her respirations are 20. Her head, eyes, ears, nose, and throat examinations are essentially normal. Inspection of her anterior and posterior chest shows no abnormalities. On auscultation of her chest, there is decreased air movement and a high-pitched whistling on expiration in all lobes. Percussion reveals resonant lungs.Which disorder of the thorax or lung does this best describe? A) spontaneous pneumothorax B) chronic obstructive pulmonary disease (COPD) C) asthma D) pneumonia

C) asthma Asthma causes shortness of breath and a nocturnal cough. It is often associated with a history of allergies and can be made worse by exercise or irritants such as smoke in a bar. On auscultation there can be normal to decreased air movement. Wheezing is heard on expiration and sometimes inspiration. The duration of wheezing in expiration usually correlates with severity of illness, so it is important to document this length (e.g., wheezes heard halfway through exhalation). Realize that in severe asthma, wheezes may not be heard because of the lack of air movement. Paradoxically, these patients may have more wheezes after treatment, which actually indicates an improvement in condition. Peak flow measurements help to discern this.

The following information is recorded in the health history: "The patient completed 8th grade. He currently lives with his wife and two children. He works on old cars on the weekend. He works in a glass factory during the week." Which category does it belong to?A. Chief complaint B. Present illness C. Personal and social history D. Review of systems

C. Personal and social history Personal and social history information includes educational level, family of origin, current household status, personal interests, employment, religious beliefs, military history, and lifestyle (including diet and exercise habits; use of alcohol, tobacco. and/or drugs; and sexual preferences and history). All of this information is documented in this example.

When holding the ophthalmoscope to examine a patient's eyes, which of the following are important parts of the technique? A) The ophthalmoscope should be held against the bony orbit of the examiner's eye. B) The examiner should instruct the patient to look up and over his/her shoulder at a point on the wall. C) The thumb of the hand not holding the ophthalmoscope should be placed across the patient's eyebrow. D) All of the above.

D) All of the above.

A 47-year-old receptionist comes to your office, complaining of fever, shortness of breath, and a productive cough with golden sputum. She says she had a cold last week and her symptoms have only gotten worse, despite using over-the-counter cold remedies. She denies any weight gain, weight loss, or cardiac or gastrointestinal symptoms. Her past medical history includes type 2 diabetes for 5 years and high cholesterol. She takes an oral medication for both diseases. She has had no surgeries. She denies tobacco, alcohol, or drug use. Her mother has diabetes and high blood pressure. Her father passed away from colon cancer. On examination you see a middle-aged woman appearing her stated age. She looks ill and her temperature is elevated, at 101. Her blood pressure and pulse are unremarkable. Her head, eyes, ears, nose, and throat examinations are unremarkable except for edema of the nasal turbinates. On auscultation she has decreased air movement, and coarse crackles are heard over the left lower lobe. There is dullness on percussion, increased fremitus during palpation, and egophony and whispered pectoriloquy on auscultation.What disorder of the thorax or lung best describes her symptoms? A) spontaneous pneumothorax B) chronic obstructive pulmonary disease (COPD) C) asthma D) pneumonia

D) pneumonia Pneumonia is usually associated with dyspnea, cough, and fever. On auscultation there can be coarse or fine crackles heard over the affected lobe. Percussion over the affected area is dull and there is often an increase in fremitus. Egophony and pectoriloquy are heard because of increased transmission of high-pitched components of sounds. These higher frequencies are usually filtered out by the multiple air-filled chambers of the alveoli.

A college student presents with a sore throat, fever, and fatigue for several days. You notice exudates on her enlarged tonsils. You do a careful lymphatic exam and notice some scattered, small, mobile lymph nodes just behind her sternocleidomastoid muscles bilaterally. What group of nodes is this? A) submandibular B) anterior cervical C) supraclavicular D) posterior cervical

D) posterior cervical

Which of the following is consistent with good percussion technique? A) allow all of the fingers to touch the chest while performing percussion B) maintain stiff wrist and hand C) leave the plexor finger on the pleximeter after each strike D) strike the pleximeter over the distal interphalangeal joint

D) strike the pleximeter over the distal interphalangeal joint

When assessing a newborn infant who is 5 minutes old, the nurse knows that which of these statements would be true? A) the L ventricle is larger and weighs more than the R ventricle B) the circulation of a newborn is identical to that of an adult C) the foramen ovale closes just minutes before birth and the ductus arteriosis closes immediately after D) there is an opening in the atrial septum where blood can flow into the L side of the heart

D) there is an opening in the atrial septum where blood can flow into the L side of the heart

Examiner's hand is placed on the patient's right knee and the patient is asked to raise his or her right thigh against the examiner's hand. This is described by: a) Rovsing's sign b) Psoas sign c) Obturator sign d) Cutaneous hyperesthesia

c) Obturator sign

Pain in the right lower quadrant during palpation of the left lower quadrant. This is described by: a) Rovsing's sign b) Psoas sign c) Obturator sign d) Cutaneous hyperesthesia

a) Rovsing's sign

Which is palpable 6 cm below the right costal margin in the midclavicular line during inspiration? a) liver edge b) spleen edge

a) liver edge

Pain elicited when the patient's right thigh is flexed at the hip with the knee bent, and the leg is internally rotated at the hip. This is described by: a) Rovsing's sign b) Psoas sign c) Obturator sign d) Cutaneous hyperesthesia

c) Obturator sign

Pain elicited by gently picking up a fold of abdominal skin anteriorly. This is described by: a) Rovsing's sign b) Psoas sign c) Obturator sign d) Cutaneous hyperesthesia

d) Cutaneous hyperesthesia

A 35-year-old accountant presents to the clinic for evaluation of sudden onset of pain in his rectum. On examination of the anus, you see a swollen, bluish, ovoid mass at the anal margin, located at the 8 o'clock position. What is your most likely diagnosis? (A) External hemorrhoid (B) Internal hemorrhoid (C) Rectal prolapse (D) Rectal polyp

(A) External hemorrhoid An external hemorrhoid is a dilation of the hemorrhoidal veins, which originate below the pectinate line and are covered with skin. They seldom produce symptoms unless thrombosis occurs. If thrombosis occurs, there is an acute onset of local pain, which is increased by defecation and by sitting. A tender, swollen, bluish, ovoid mass may be visible at the anal margin.

A 28-year-old male waiter presents to your office complaining of a growth on the back of his left hand. He states it started slowly months ago but has now enlarged enough to be embarrassing. He states it only hurts when he is holding a tray of food up in the air with his left hand. He has no significant past medical history and no one else in the family has this problem. On exam, you 15-3 note a 2-cm round cystic-like lesion on the dorsum of his left wrist over the carpals. It is more prominent when he flexes his wrist. What is the most likely cause of his hand swelling? (A) Ganglion (B) Chronic tophaceous gout (C) Acute rheumatoid arthritis (D) Heberden's nodes

(A) Ganglion A ganglion is a growth along the tendon sheaths of joint capsules. Ganglions generally appear in young adults and are usually nontender. They are more prominent when the wrist is flexed.

A 26 year old woman presents for eval of hair loss. She also has noticed increased weight loss and diarrhea. You diagnose Grave's disease (hyperthyroidism). On physical examination of the hair, what would you expect to find? (A) Fine texture (B) Coarse texture (C) Oily hair (D) Dry hair

(A) fine texture

You notice that your patient's submental lymph nodes are enlarged. You would then assess the: (A) Supraclavicular area (B) Infraclavicular area (C) Area proximal to the enlarged node (D) Area distal to the enlarged node

(C) Area proximal to the enlarged node When nodes are abnormal, check the area from which they drain.

A 36-year-old computer programmer presents to the office with pain with urination, and fever. On palpation of the prostate, his gland is swollen, tender, and warm to the touch. Your most likely diagnosis is? (A) Normal prostate (B) Benign prostatic hyperplasia (C) Prostate cancer (D) Prostatitis

(D) Prostatitis In acute prostatitis, the patient typically has fever and dysuria. The gland is very tender, swollen, firm, and warm to touch.

A 15-year-old high-school student presents to your office for a sports physical He is 65 inches in height and weighs 250 pounds. His BMI falls in which category? (A) Underweight (B) Normal weight (C) Overweight (D) Obese

(D) Obese The patient is obese. Obesity is defined as a BMI over 30. This patient has a calculated BMI of 43. Underweight is defined as a BMI of less than 17; normal weight is defined as a BMI between 17 and 23; and Overweight is defined as a BMI between 23 and 30. Calculation of BMI: weight in kilograms divided by height in meters squared.

A 25-year-old swim instructor presents to your clinic complaining of an itchy scalp. You diagnose seborrheic dermatitis. What physical findings are most consistent with this diagnosis? (A) Erythema of the scalp (B) Pustules on the scalp (C) Dry, flaking areas on the scalp (D) Ecchymoses on the scalp

(C) Dry, flaking areas on the scalp

You are in the emergency room assessing a patient with abdominal pain and fever. You are performing an abdominal examination to assess for peritoneal signs. Which one of the following is NOT a peritoneal sign? (A) Rebound tenderness (B) Involuntary guarding (C) Rigidity of the abdomen (D) Voluntary guarding

(D) Voluntary guarding

A 68-year-old retired farmer comes to your office for evaluation of a skin lesion. On the right temporal area of the forehead, you see a flattened papule the same color as his skin, covered by a dry scale that is round and feels hard. He has several more of these scattered on the forehead, arms, and legs. Based on this description, what is your most likely diagnosis? A) Actinic keratosis B) Seborrheic keratosis C) Basal cell carcinoma D) Squamous cell carcinoma

A) Actinic keratosis This is a typical description of actinic keratosis. Actinic keratosis may be easier to feel than to see. If left untreated, approximately 1% of cases can develop into squamous cell carcinoma.

A 35-year-old archaeologist comes to your office (located in Phoenix, Arizona) for a regular skin check-up. She has just returned from her annual dig site in Greece. She has fair skin and reddish-blonde hair. She has a family history of melanoma. She has many freckles scattered across her skin. From this description, which of the following is not a risk factor for melanoma in this patient? A) Age B) Hair color C) Actinic lentigines D) Heavy sun exposure

A) Age The risk for melanoma is increased in people over the age of 50; our patient is 35 years old. The other answers represent known risk factors for melanoma. Especially with a family history of melanoma, she should be instructed to keep her skin covered when in the sun and use strong sunscreen on exposed areas.

Another name for deep tendon reflexes is: A) muscle stretch reflexes B) hyperresponsive reactions C) febrile seizure D) none of the above

A) muscle stretch reflexes

A 69-year-old woman presents to your office with the complaint of pain. She was diagnosed with breast cancer that has metastasized to the bone 6 months ago. At the time, she did not want radiation therapy even for relief of the pain. You then discussed with her that her prognosis is poor and that her time to live is limited. She was not interested in hearing about it. You observe that she has lost more weight and that she is even more anorexic than at her previous visit 1 month ago. Today, she is frustrated because she hurts when she gets up and tries to walk around. She has planned a trip to the Bahamas to take place in 4 more months. What stage of grief is this patient currently in? (A) Denial (B) Anger (C) Bargaining (D) Depression or sadness (E) Acceptance

(A) Denial This patient has not yet accepted the fact that she is dying. She is making plans for the future that seem inappropriate given the diagnosis, prognosis, and her overall physical decline. The patient does not seem angry. She has not expressed anger about her condition or about the "unfairness" of her prognosis, nor toward the clinician for the diagnosis has been made. This scenario does not illustrate any evidence of bargaining. The patient has not expressed sadness nor depression. She has clearly not accepted her diagnosis.

Dakota is a 14-year-old boy who just noticed a rash at his ankles. There is no history of exposure to ill people or other agents in the environment. He has a slight fever in the office. The rash consists of small, bright red marks. When they are pressed, the red color remains. What should you do? A) Prescribe a steroid cream to decrease inflammation. B) Consider admitting the patient to the hospital. C) Reassure the parents and the patient that this should resolve within a week. D) Tell him not to scratch them, and follow up in 3 days.

B) Consider admitting the patient to the hospital. Although this may not be an impressive rash, the fact that they do not "blanch" with pressure is very concerning. This generally means that there is pinpoint bleeding under the skin, and while this can be benign, it can be associated with life-threatening illnesses like meningococcemia and low platelet counts (thrombocytopenia) associated with serious blood disorders like leukemia. You should always report this feature of a rash immediately to a supervisor or teacher.

A normal prostate feels: A) hard and nodular B) hot and boggy C) rounded and heart shaped D) none of the above

C) rounded and heart shaped

You are seeing an elderly man with multiple complaints. He has chronic arthritis, pain from an old war injury, and headaches. Today he complains of these pains, as well as dull chest pain under his sternum. What would the order of priority be for your problem list? A) Arthritis, war injury pain, headaches, chest pain B) War injury pain, arthritis, headaches, chest pain C) Headaches, arthritis, war injury pain, chest pain D) Chest pain, headaches, arthritis, war injury pain

D) Chest pain, headaches, arthritis, war injury pain

When palpating a patient's neck you feel bilateral soft, mobile, less than 1cm nodule located just below the angle of the mandible. You recognize this as what lymph node? a) sub-mandibular b) anterior cervical c) tonsillar d) posterior cervical

c) tonsillar

A patient presents to you with shoulder pain after falling during an ice storm. On examination, he exhibits localized shoulder pain when you perform the "crossover test" (arm moved across the chest toward the opposite side while extended at the elbow). Which of the following is the most likely site of injury? a.Rotator cuff b.Bicipital tendon c.Glenohumeral joint d.Acromioclavicular joint

d.Acromioclavicular joint Localized tenderness or pain with adduction suggests inflammation of the acromioclavicular joint

A 21-year-old female presents to your clinic for her annual exam. She informs you that for the last 3 months, she has felt a lump in her left breast above the nipple. She has had no discharge from the nipple. Her past medical history is nonsignificant. She has not been sexually active and is on no medication. She denies any tobacco, alcohol, or drug use. Her paternal grandmother had breast cancer in her seventies and did well with surgery and radiation. On physical exam, you feel a firm disc like lump at 12 o'clock on the left breast. It is easily delineated, mobile, and is nontender. The patient only has shoddy nodes in the left axilla. There are no unusual skin changes. The right breast and the remainder of her heart, lung, abdominal, and pelvic exam are unremarkable. What form of breast mass is she most likely to have? (A) Fibroadenoma (B) Cysts (C) Cancer

(A) Fibroadenoma Fibroadenomas often start during puberty and young adulthood. They may be single or multiple and are often round or disc-like. They can be soft but are usually firm. They are very mobile and delineated from the surrounding tissue. They are generally non-tender and show no signs of skin retraction.

What is the preferred order for examination of the abdomen? (A) Inspection, auscultation, percussion, palpation (B) Percussion, auscultation, palpation, inspection (C) Auscultation, inspection, palpation, percussion (D) Inspection, palpation, auscultation, percussion

(A) Inspection, auscultation, percussion, palpation

A 75-year-old widow presents to your office complaining of a painful black spot to her left third toe. She states that it started smaller but has grown over the last few days. Her past medical history is significant for an acute myocardial infarction 6 years ago, with three-vessel bypass surgery afterwards. She has also had high blood pressure for 15 years and was diagnosed with type-2 diabetes last fall. She denies any tobacco, alcohol, or drug use. On review of systems, she does note that she is unable to walk a half-mile anymore without needing to stop due to pain in her legs. On exam, you find an elderly patient in no acute distress. She has no swelling or edema in her legs, although the skin color is fairly red. You can palpate a weak posterior tibia pulse but no dorsal pedis pulses. The toe in question is black the last distal centimeter. What peripheral vascular disorder is most likely the cause of her ulcer? (A) Insufficiency ulcer (B) Venous insufficiency ulcer (C) Arterial neuropathic ulcer

(A) Insufficiency ulcer Ulcers from a decrease in arterial blood flow are often found at the distal portion of the extremities, such as the toes. The foot is generally red in color with no swelling. Pulses are often weak or absent.

A 25-year-old celebrity with a known history of intravenous drug use presents to the emergency room for evaluation of a 5-day history of nausea, emesis, and right-upper-quadrant abdominal pain. On general survey, he appears ill and his skin is distinctly yellow. He has a temperature of 102.5°F and a heart rate of 112 bpm. You provisionally diagnose him with acute hepatitis. What would you expect to find on abdominal examination? (A) Liver edge is tender and 4 to 5 finger-breadths below the RCM (B) Liver edge is nonpalpable (C) Liver edge is tender and 1 finger-breadth below the RCM (D) Liver edge is nontender and 4 to 5 finger-breadths below the RCM

(A) Liver edge is tender and 4 to 5 finger-breadths below the RCM

During your examination of a woman in her second trimester of pregnancy, you note the presence of a small amount of yellow drainage from the nipples. You know that this is: (A) Most likely to be colostrum and considered a normal finding this late in her pregnancy (B) A sign of breast cancer (C) Too early in the pregnancy for lactation to begin; the woman needs a referral to a specialist (D) An indication that the woman's milk is forming

(A) Most likely to be colostrum and considered a normal finding this late in her pregnancy During the second trimester, colostrum, the precursor of breast milk, may appear. Colostrum is yellow in color and contains more minerals and protein, but less sugar and fat than breast milk.

A 48-year-old unemployed man comes into the urgent care clinic complaining of increased swelling in his lower extremities. He states that he has had swelling for a number of months but that the swelling worsened last night. He complains of no ulcers or pain in the legs. His medical history reveals a past diagnosis of "liver problems." He isn't sure what the exact diagnosis was. He states he used to drink a case of beer a night but he has recently cut back to a six-pack. He has smoked one pack of cigarettes a day for over 30 years and denies any IV drug use. On exam, you note that both legs are swollen, with no skin thickening or ulcerations. Pressing your thumb against his tibia reveals soft edema. Auscultation of his lungs reveals no crackles. His heart sounds are regular, with no murmurs, rubs, or gallops. His abdomen reveals a small liver but no discernable fluid wave. What diagnosis of peripheral causes of edema is most likely the cause of his problem? (A) Pitting edema (B) Chronic venous insufficiency (C) Lymphedema

(A) Pitting edema Pitting edema is the result of an interstitial leak from the vessels due to hypervolemia or from a decrease in oncotic pressure of the fluid, allowing fluid to leave the vessels. In this case, probable cirrhosis of the liver leads to malnutrition, resulting in low albumin, which is causing the oncotic leak. The swelling in the extremities is always bilateral, with generally no skin thickening or ulcerations.

A 7-year-old elementary school child presents with her mother to the clinic for evaluation of itching. They deny any exposure to new soaps or detergents, pollens or grasses, or new foods. On physical examination, there are small papules, pustules, lichenified areas, and excoriations. Using a magnifying lens, you identify a burrow on the finger webs. What is the most likely diagnosis? (A) Scabies (B) Atopic dermatitis (C) Acne (D) Telangiectasias

(A) Scabies The presence of a burrow is most diagnostic of scabies. A burrow is a minute, slightly raised tunnel in the epidermis commonly found on the finger webs and on the sides of the fingers. The patient experiences intense itching, which results in excoriations and lichenified areas secondary to the scratching.

An 18-year-old college student presents to the clinic with the complaint that her heart is "racing." You obtain blood for thyroid studies and diagnose her with Graves' disease (hyperthyroidism). On physical examination of her eyes, what would you expect to see? (A) Recession (B) Protrusion (C) Clouding of the cornea (D) Ciliary injection

(B) Protrusion

The following finding is considered normal when assessing an aging male: (A) A decrease in scrotal color (B) Enlargement of the testes and scrotum (C) A decrease in the size of the penis (D) The presence of a hydrocele, or fluid in the scrotum

(C) A decrease in the size of the penis In the older male, you may note thinner, more sporadic gray pubic hair and a decrease in the size of the penis. A change in color, size, and scrotal fluid are symptoms requiring further investigation and are not associated with aging.

A 55-year-old retired woman comes to your office complaining of worsening problems with walking accompanied by falling down. She states that she has to watch the ground while she is walking or she will often misstep and fall. She has a 20-year history of type-2 diabetes and has had a history of diabetic foot ulcers in the past. She denies any smoking, drug, or alcohol use. On exam, you note that she has decreased feeling on the planter surfaces of her feet. She cannot remain steady with her feet together and her eyes closed; she remains steady with her eyes open. While watching her walk, you also note that she has a wide gait with slapping motion with her feet on the floor. What abnormalities of gait best describe her symptoms and signs? (A) Sensory ataxia (B) Cerebellar ataxia (C) Parkinsonian gait (D) Steppage gait

(A) Sensory ataxia Sensory ataxia is associated with loss of position of the feet and legs often due to peripheral neuropathies such as those associated with diabetes. The gait is generally wide based and unsteady. The patients usually will throw their feet down heavily forward and outward. They must also watch the ground for guidance and usually cannot stand steady with their feet together (positive Rhomberg sign).

A 68-year-old tax attorney is in the hospital with a stroke. He has residual left-sided paralysis and is bed-ridden. On examination of the skin, you notice an area on his sacrum that is red with a boggy consistency, and you diagnose a pressure ulcer. What stage is this pressure ulcer? (A) Stage I (B) Stage II (C) Stage III (D) Stage IV

(A) Stage I The patient has a stage I ulcer, which is defined as intact skin that has a change in temperature, consistency, sensation, or color. The temperature can be warm or cool; the consistency can be firm or boggy; the patient may experience pain or itching; the color may be red, blue, or purple.

A 55-year-old disabled Vietnam veteran comes into the ER complaining of severe pain in his left arm, which occurred suddenly approximately 2 hours ago. He also states that the arm is numb and feels cold to him from the elbow down. He has had similar episodes in the past with his legs, which required amputation at the mid-thigh bilaterally. He has a 20-year history of hypertension and a 60-pack-year history of smoking. He is a recovering alcoholic but denies any drug use. His father passed away from a stroke at the age of 65. On exam, he is anxious and appears to be in a great deal of pain. The left arm appears pale and cool to the touch. No peripheral pulses in either the radial or ulna arteries are palpated. The pulses also cannot be elicited by Doppler. Reflexes in the brachioradialis tendon are decreased. He has decreased strength in his wrist and hand and cannot discern two-point discrimination. What peripheral vascular disorder best describes his situation? (A) Artherosclerosis obliterans (B) Acute arterial occlusion (C) Deep venous thrombosis (D) Cellulitis

(B) Acute arterial occlusion The throwing of an embolism, or thrombosis, causes sudden symptoms associated with a cold, pale, pulseless extremity.

A 35-year-old male athlete comes into the ER complaining of severe lower abdominal pain and vomiting. He relates the pain began several hours ago after he cycled at the gym and lifted weights. He states he has had occasional lower abdominal pain with lifting weights in the past but nothing like this. He has no significant past medical history and denies any tobacco, drug, or alcohol use. On exam, you find a young man who appears very ill. He is pale and vomiting clear green liquid. As you examine him, his emesis becomes darker and malodorous. On exam, he has high-pitched increased bowel sounds and has voluntary guarding and rebound. His testicles are descended bilaterally, and his right testicle seems tender and enlarged. Auscultating it, you also hear bowel sounds. What diagnosis is most likely the cause of his pain? (A) Acute pancreatitis (B) Acute mechanical intestinal obstruction (C) Acute cholecystitis (D) Mesenteric ischemia

(B) Acute mechanical intestinal obstruction Mechanical obstructions are often cause by tumors, surgical adhesions, and, in this case, hernia. Often straining will cause a weakened inguinal area to herniated, leading to an incarcerated hernia. Symptoms include bilious vomiting, which can become fecal in material, and the development of a rigid abdomen.

A 39-year-old seamstress presents to the ER complaining of severe upper abdominal pain that started about 2 hours ago. She had eaten a cheeseburger and fries 45 minutes before the pain began. She tried taking some calcium stomach tablets, but they didn't help. She states the pain is a 10 on a 10-point scale and is starting to hurt in her back. She says she is nauseated and needs to vomit. She does admit to having had milder pains similar in the past but never to this extent. Her past medical history is significant for high blood pressure and two caesarean sections. Her older sister and mother have had to have gallbladder surgery. She denies any tobacco, drug, or alcohol use. Her review of systems is noncontributory. On examination, you find a mildly obese woman in severe distress. She is lying on the stretcher, but she cannot find a comfortable position. Her blood pressure is mildly elevated at 140/85, and her heart rate is 110. Her temperature is normal. Her abdominal exam reveals normal bowel sounds, but she is tender in the right upper quadrant. She has a positive Murphy's sign. The remainder of her abdominal exam is normal. Her rectal is heme negative, and her pelvic exam is normal. What etiology of abdominal pain is most likely causing her symptoms? (A) Peptic ulcer disease (B) Biliary colic (C) Acute cholecystitis (D) Acute pancreatitis

(B) Biliary colic Biliary colic is caused by a sudden obstruction in the cystic duct or the common bile duct by a gallstone. A fatty meal can precipitate the attack. The pain is usually severe and steady and can radiate to the scapula. Nausea, vomiting, and restlessness are common. A positive Murphy's sign is often found.

A 31-year-old nurse presents to your office complaining about a painful lump in her right breast next to the nipple. She states it has been there for several months and won't go away. She recently started taking birth control pills after remarrying. Her past medical history is significant for asthma and two spontaneous vaginal deliveries. Family history reveals no breast or ovarian disorders. She denies any tobacco or drug use and drinks alcohol socially. On exam, you feel a 1-cm lump medial to her areola. It is round soft and very tender. There are no skin retractions and no palpable lymph nodes in the axilla. Her left breast also has a ½-cm soft lump medial to the areola that is not quite as tender. The patient states she hadn't noticed that area before. What type of breast mass is the most likely cause of her pain? (A) Fibroadenoma (B) Cysts (C) Cancer

(B) Cysts Cysts often start in women between 30 and 50 years of age. They tend to be round but can be soft or firm. They are mobile and well delineated. Generally they are tender, and they can change in size during the menstrual cycle. They become more pronounced in some women on oral contraceptives. Careful evaluation is still warranted due the rare possibility of breast cancer in this age group.

A 71-year-old woman is brought to the ER by her family, who state that she is not acting like herself. They say that several hours ago she began to slur her words as she started wandering around the house. They heard her calling to her husband, who has been deceased for over 10 years. After several minutes of this, she said she wasn't feeling well and laid down on the sofa; 16-5 she became sleepy. When they tried to ask her what was wrong, she snapped at them. Although the woman is retired, she is very active in her church and senior citizens club. Her past medical history is significant for 3 vaginal deliveries, 10 years of hypertension, and 5 years of type-2 diabetes. She does not smoke, drink alcohol, or take drugs. Review of systems reveals that she had mentioned to her daughter the day before that urinating is painful and that her back is sore. On exam, you find an elderly woman who is lethargic. You have to speak loudly for her to answer questions. She is alert to person but not place or time. Her vital signs and exam of the head, heart, lungs, and abdomen are all normal. She is tender at the costal angle on the right. What is the most likely cause of her mental status change? (A) Psychotic reaction to major depression (B) Delirium (C) Dementia

(B) Delirium The hallmark of delirium is that it occurs acutely. The course can be fluctuating, and the level of consciousness is disturbed. The patient can become somnolent, as in this case, or agitated. Illusions and hallucinations are common. Patients are often disoriented to place and time but not to person. Metabolic changes, poisonings, sepsis, and delirium tremens are causes. This episode probably began as a urinary tract infection that became uroseptic.

A 35-year-old swimming teacher presents to the office for an annual skin examination. She has several tan, flat, round and oval lesions with sharply defined borders. The lesions are less than 6 mm in diameter. She has one lesion that is dark, 6.5 mm, and has an irregular border, which fades into the surrounding skin. It is not elevated. What is your most likely diagnosis of this lesion? (A) Benign nevus (B) Dysplastic nevus (C) Seborrheic keratosis (D) Actinic keratosis

(B) Dysplastic nevus A dysplastic nevus is varied in color, but often dark. It is larger than 6 mm and has irregular borders, which fade into the surround skin. Without further intervention, it is difficult to determine whether this patient's lesion is an early malignant melanoma or not.

A 14-day-old infant presents to the clinic for a follow-up visit from the newborn nursery. On examination, she has yellow-white pustules surrounded by a red base, which are scattered diffusely over the face and trunk. What is the most likely diagnosis for this rash? (A) Neonatal acne (B) Erythema toxicum (C) Seborrhea (D) Jaundice

(B) Erythema toxicum Erythema toxicum consists of yellow or white pustules, which are surrounded by a red base. The rash is scattered diffusely over the infant's body and frequently changes location. The palms and soles are often spared.

A 68-year-old homemaker presents to your clinic complaining of difficulty swallowing. It began several months ago when she would attempt to eat steak. Over time, it has progressed to the point that she can only sip on soups and broths. She has sometimes had to regurgitate her food back up to get relief from the pain in her chest when she attempts to eat. Her past medical history is significant for diet controlled, type-2 diabetes. She had a 60-pack-year smoking history but quit smoking 15 years ago. She denies any drug or alcohol abuse. On review of systems, she has lost approximately 15 lbs in the past 3 months. She denies any heartburn, constipation, or diarrhea. On exam, you find a pleasant elderly woman in no acute distress. Her cardiac, pulmonary, and abdominal exams are all normal. What disorder of dysphagia is the most likely cause for her symptoms? (A) Esophageal stricture (B) Esophageal cancer (C) Esophageal spasm (D) Scleroderma

(B) Esophageal cancer Esophageal cancer often is slowly progressive but invariably leads to difficulty in swallowing liquids. Weight loss often develops as the disease progresses. Risk factors include tobacco and alcohol use.

You are palpating the abdomen of a 35-week pregnant woman and note that the fetal head is facing downward toward the pelvis. You would document this as: (A) Fetal lie (B) Fetal presentation (C) Fetal attitude (D) Fetal variety

(B) Fetal presentation Fetal presentation describes the part of the fetus that is entering the pelvis first. Fetal lie is orientation of the fetal spine to the maternal spine. Attitude is the position of the fetal parts in relation to each other, and fetal variety is the location of the fetal back to the maternal pelvis.

A 22-year-old social worker comes into the clinic for a routine checkup. His blood pressure is 155/102. In each of his previous visits, including one 2 weeks ago for treatment of an upper respiratory infection, his blood pressure has been 130s/80s. He has no family history of high blood pressure. He has no symptoms of headache, blurred vision, or chest pain. What is the most likely reason for his blood pressure being elevated today? (A) He sat down for 10 minutes prior to this measurement being obtained (B) He drank a cup of coffee 10 minutes prior to his blood pressure measurement (C) His arm was bare when the blood pressure was measured (D) He was seated and his arm was supported a little above his waist when the measurement was obtained

(B) He drank a cup of coffee 10 minutes prior to his blood pressure measurement Caffeine ingested within 30 minutes of checking a blood pressure measurement can artificially increase the patient's blood pressure reading.

When performing a genitourinary assessment, you note that the urethral meatus is positioned ventrally. This is most likely to be: (A) A stricture (B) Hypospadius (C) The result of a circumcision (D) Related to the aging process

(B) Hypospadius Hypospadius is a congenital displacement of the urethral meatus to the inferior surface of the penis. A groove extends from the displaced urethral meatus to its normal location on the tip of the glans. There is no association between aging and displacement of the urethral meatus.

A 55-year-old postal worker presents to the emergency room with chest pain. On a scale of 1 to 10, he rates the pain a 9. It started 30 minutes ago and hasn't gone away. He has never had it before. He denies any injury or trauma; he has no medical conditions that are being treated. Nothing makes the pain better or worse. When questioned further, he has noticed nausea and increased sweating with the pain. Based on this description, which of the seven attributes of this symptom have not been addressed? (A) Remitting or exacerbating factors (B) Quality (C) Severity (D) Timing

(B) Quality Quality is the only one of the seven attributes of a symptom listed that is not addressed. The patient has not described the kind of pain he is experiencing other than that it is in his chest. Pain may be an ache, a sharp sensation, a pressure-like sensation, or a burning sensation, which are all important descriptors in helping to determine your differential diagnosis.

A 19-year-old male college student presents to your clinic the week following spring break complaining of 1-day abdominal pain and watery diarrhea. He thinks he has seen blood mixed with his stool. He has had some nausea but no vomiting. He had recently traveled to Central America over the break to go scuba diving. His past medical history is not significant. He drinks six to eight beers during the weekend but denies any tobacco or drug use. On review of symptoms, he does have night sweats and rigors. On exam, you find he is febrile at 102.3°F and he is tachycardic at 115 bpm. Otherwise, his heart and lung exam are normal. Upon auscultation, he has increased bowel sounds, and on palpation, he is diffusely tender. He has no rebound or guarding, but his rectal is guaiac positive. What cause of diarrhea is most likely? (A) Secretory infections (B) Inflammatory infections (C) Irritable bowel syndrome (D) Malabsorption syndrome

(B) Inflammatory infections Bloody diarrhea with a fever after foreign travel is often indicative of invasion of the intestinal mucosa by enteropathic bacteria. An in-depth travel history of different foods and liquids is often warranted

A 24-year-old housewife and mother presents to the clinic for evaluation of rectal bleeding. She has intermittent constipation and frequently has to use a laxative to have a bowel movement. On examination of the anus, you see a reddish, moist, protruding mass located at the 5 o'clock position. What is your most likely diagnosis? (A) Hemorrhoid (B) Internal hemorrhoid (C) External rectal prolapse (D) Rectal polyp

(B) Internal hemorrhoid An internal hemorrhoid is an enlargement of the normal vascular cushions that are located above the pectinate line. During defecation, they may cause bleeding. If they prolapse, they are visible as a reddish, moist, protruding mass.

A 13-year-old boy is brought into the urgent care clinic by his father with a complaint of swelling around his right elbow. He is in 8th grade football and has been lifting weights, despite having just started puberty. He has no significant past medical history and recalls no specific incident of trauma to the elbow. On exam, the patient has a soft swelling around the posterior of the elbow. He has no redness or warmth and minimal pain. He is nontender over the bony prominences of the ulna, radial head, and humorous. What disorder of the elbow is the most likely diagnosis? (A) Arthritis (B) Olecranon bursitis (C) Epicondylitis (D) Rheumatoid nodules

(B) Olecranon bursitis Sudden and repetitive trauma often causes swelling and inflammation in the bursa. In this case, the repetitive injury of inappropriate weightlifting led to the accumulation of fluid in the bursa sac.

A 28-year-old bus driver presents to the emergency room for pain and drainage from an area at the bottom of her spine. This has never occurred before. On examination of the rectum, a small opening is identified in the midline superficial to the lower sacrum. There is a surrounding halo of erythema. It is tender to palpation and purulent material is expressed. Your most likely diagnosis of this condition is? (A) Anorectal fistula (B) Pilonidal cyst (C) Anal fissure (D) Thrombosed external hemorrhoid

(B) Pilonidal cyst A pilonidal cyst is identified by the opening of a sinus tract on the skin. It is usually located in the midline superficial to the coccyx or the lower sacrum. There may be a small tuft of hair at the opening, and it may be surrounded by a halo of erythema.

A 43-year-old female kindergarten teacher presents to your office with the complaint of fainting this morning. She reports this occurred after standing up from her bed after having been sick all night with diarrhea and vomiting. She has had no fever and is unable to keep fluids down. She has had no chest pain, shortness of breath, or palpitations. Her past medical history is significant for two spontaneous vaginal deliveries and a 10-year-history of hypothyroidism. On exam, she has a fast heart rate of 110 bpm. Her abdomen is tender, with no signs of a surgery. What is the most likely underlying cause of her fainting? (A) Vasodepressor syncope (B) Postural (orthostatic) hypotension (C) Cardiac arrhythmia (D) Hysterical faint

(B) Postural (orthostatic) hypotension Orthostatic hypotension can often be due to hypovolemia, which in this case is from vomiting and diarrhea. It generally occurs upon standing up after being in a recumbent position.

A 19-year-old male college student presents to your clinic with a complaint of tremors in his hands. He states that it embarrasses him in front of his friends. He notices it only when he isn't doing anything with his hands; it gets better when he drinks alcohol. He has no significant past medical history but does relate that his older sister and mother have had similar problems. Upon exam, he holds his hands out in front of him revealing a fine tremor. There are no additional movements with his fingers. When he does purposeful movement, the tremors disappear. What involuntary movement disorder best describes his symptoms and signs? (A) Resting tremors (B) Postural tremors (C) Intention tremors (D) Athetosis

(B) Postural tremors Postural tremors occur when a person is holding the affected part of his body in a postural position. These tremors are often associated with hyperthyroidism, anxiety, and benign familial essential tremors, such as is the case here. Intentional movement can either worsen or improve the tremors.

To correctly insert the speculum: (A) Instruct the woman to bear down, open the speculum blades and apply in a swift upward movement (B) Press the inferior margin of the introitus down to enlarge the vaginal opening, open, insert the speculum with the blades at an oblique downward angle, and apply gentle downward pressure (C) Insert the blades of the speculum on a horizontal plane, turning them to a 45° angle as you continue to insert them; ask the woman to bear down to ease insertion (D) Lock the blades open by turning the thumbscrew. Once you have them open, apply pressure to the introitus and insert the blades at a 45° angle downward to bring the cervix into view

(B) Press the inferior margin of the introitus down to enlarge the vaginal opening, open, insert the speculum with the blades at an oblique downward angle, and apply gentle downward pressure Hold the speculum in your right hand with the index and the middle fingers surrounding the blades and your thumb under the thumbscrew. This prevents the blades from opening painfully during insertion. With your left index and middle fingers, push the introitus down and open to relax the pubococcygeal muscle. Tilt the width of the blades at an oblique angle and insert the speculum past your left fingers, applying any pressure downward. This avoids pressure on the sensitive urethra above it. Ease insertion by asking the woman to bear down. This relaxes the perineal muscles and opens the introitus. As the blades pass your left fingers, withdraw the fingers. Now turn the width of the blades horizontally and continue to insert at a 45° angle downward toward the small of the woman's back. This matches the angle of the vagina vault.

Correct instructions about the self-breast examination (BSE) are: (A) Tell the woman that the best time to perform BSE is in the middle of the menstrual cycle (B) Recommend that the best time to perform BSE is 4-7 days after the first day of the menstrual period (C) Tell the woman that if she is pregnant, she should not perform BSE until the baby is born (D) Urge the woman that she needs to do BSE bimonthly unless she has fibrocystic breast tissue

(B) Recommend that the best time to perform BSE is 4-7 days after the first day of the menstrual period Help each woman establish a regular schedule of self-care. The best time to conduct breast self-examination is right after the menstrual period, or the fourth through the seventh day of the menstrual cycle, when estrogen stimulation is low and the breasts are least congested. Advise the pregnant woman or menopausal woman who is not having menstrual periods to select a familiar date to examine her breasts each month, for example, her birth date or the day the rent is due.

The muscles in the neck that are innervated by cranial nerve XI are: (A) Spinal accessory and omohyoid (B) Sternomastoid and trapezius (C) Trapezius and sternomandibular (D) Sternomandibular and spinal accessory

(B) Sternomastoid and trapezius The major anterior neck muscles are the trapezius and sternomastoid. They form two triangles on the side of the neck, which can be used as landmarks for lymph nodes and other vessels.

A 22-year-old female college student presents to your clinic for a post-hospitalization follow-up. She was admitted last week for appendicitis and had an appendectomy. She states that she is having no abdominal pain but is having pain on the back of her left wrist. She states that 2 days after she was discharged the area became painful, red, and swollen. She does think there was an IV line there, but she isn't sure. Her past medical history is non-significant and she denies any tobacco, drug, or alcohol use. On exam, the dorsum of her left hand is tender, warm, and red, with some minimal swelling. On palpation, you feel a hard knot 2 cm by 3 cm long. What is the peripheral vascular disorder most likely causing her problem? (A) Raynaud's disease (B) Superficial thrombophlebitis (C) Acute arterial occlusion (D) Deep venous thrombosis

(B) Superficial thrombophlebitis Superficial thrombophlebitis is caused by the clotting of superficial veins; it is often due to trauma. There is localized redness, swelling, warmth, and pain. It can be accompanied by fever.

A 55-year-old legal secretary comes into your office to establish care. She starts the interview by telling you her whole life story, tells you about her husband, children, and grandchildren, and then pulls out a written list of 20 different items that relate to her health that she wants to address with you in today's interview. She is very friendly and relaxed. What specific type of patient is this? (A) Anxious patient (B) Talkative patient (C) Angry or disruptive patient (D) Silent patient

(B) Talkative patient This patient wants to tell you her life's story. Remember to let this type of patient talk for approximately 5-10 minutes while you listen closely to how she expresses her ideas, note the use of nonverbal cues, and so forth. A helpful technique is to have the patient focus on the most important thing that concerns her at today's visit and to schedule a follow-up appointment, setting a specific time limit on the follow-up as well.

A 20-year-old college junior presents to the clinic for a physical; he is going to be a camp counselor for the summer. He is healthy and exercises regularly. His BMI is 22. You take his blood pressure and obtain a value of 160/100. Which of the following is the most likely reason for the unexpected blood pressure measurement? (A) The width of the inflatable bladder of the cuff was 40% of the upper arm circumference (B) The length of the inflatable bladder was 50% of the upper arm circumference (C) The aneroid sphygmomanometer was calibrated just prior to measuring this patient's blood pressure (D) The patient had sat for 15 minutes prior to the measurement of his blood pressure

(B) The length of the inflatable bladder was 50% of the upper arm circumference The length of the inflatable bladder should be 80% of the upper arm circumference; if it is too short, then the blood pressure reading will be artificially increased.

A newborn infant has a 1-minute Apgar score of 6. Which of the following statements is correct? (A) This infant is normal (B) This infant has some nervous system depression (C) This infant has severe depression and requires immediate resuscitation (D) This infant is at high risk for subsequent central nervous system and other organ system dysfunction

(B) This infant has some nervous system depression An Apgar score that is between 5 and 7 at 1 minute of life indicates some nervous system depression. A normal Apgar score at 1 minute of life is between 8 and 10. An Apgar score of 0-4 at 1 minute of life indicates severe depression, requiring immediate resuscitation. An Apgar score at 5 minutes of life that is between 0 and 7 indicates that the infant is at high risk for subsequent central nervous system and other organ system dysfunction.

A 37-year-old female bank teller presents to your office complaining about accidentally urinating on herself. She relates this has happened several times in the last month. She states she realizes that she has to go to the bathroom, but before she can make it to the toilet, she soils herself. Upon further questioning, it is revealed that for years she has had to urinate frequently during the day and at night. She had thought these were bladder infections, but the urine lab tests were always normal. Her past medical history is significant for migraines and depression. She denies any tobacco or drug use. She drinks alcohol socially several times a year. On review of systems, she relates pelvic pain with sexual intercourse. On exam, she has a normal abdominal, rectal, and pelvic exam. Her bladder is appropriate in size, and her urinary analysis is negative for blood or signs of infection. Which type of urinary incontinence does she have? (A) Stress incontinence (B) Urge incontinence (C) Overflow incontinence

(B) Urge incontinence Strong detrussor contractions of the bladder overcome normal urethral resistance, causing urge incontinence. In this patient's case, a condition called interstitial cystitis has caused a deconditioning of voiding reflexes. Frequency, urgency, and voiding small amounts of urine are also often associated with urge incontinence.

A 2-year-old child presents for his annual checkup. You obtain a history and perform a physical examination. All of the findings are normal. When you do a visual acuity test, what is your most likely finding? (A) Eyes converge (B) -20/200 (C) 20/40 (D) 20/30

(C) 20/40 Children under 4 years of age have an expected visual acuity of 20/40. Eyes converging is an expected finding in children aged 0-3 months. This patient is 2 years old. The second response (-20/200) is incorrect; this is an expected finding in children who are 12 months of age. The fourth response (20/30) is likewise incorrect. Children over the age of 4 have an expected visual acuity of 20/30.

When palpating the fundus, you know that: (A) Fundal height is usually less than the number of weeks gestation, unless there is an abnormal condition such as too much amniotic fluid present (B) The fundus should be hard and slightly tender to palpation during the first trimester (C) After 20 weeks gestation, the number of centimeters should approximate the number of weeks gestation (D) Fetal movement should be felt by the examiner at the beginning of the second trimester

(C) After 20 weeks gestation, the number of centimeters should approximate the number of weeks' gestation After 20 weeks, the number of centimeters should approximate the number of weeks gestation. Also, at 20 weeks the examiner may feel fetal movement, and the head can be balloted.

When performing a genitourinary assessment on an adolescent male you notice a swelling in the scrotum, which increases with increased intraabdominal pressure and decreases when lying down. The young man complains of pain when straining. This history best describes: (A) An incisional hernia (B) A direct inguinal hernia (C) An indirect inguinal hernia (D) A femoral hernia

(C) An indirect inguinal hernia An indirect hernia generally causes pain with straining and a soft swelling in the scrotum that enlarges with increased intraabdominal pressure. It originates above the inguinal ligament near the midpoint and touches the examining finger during examination. This is the most common type of hernia, found in all ages and both sexes.

A 16-year-old sophomore presents to the clinic for a sports physical; she is accompanied by her mother. She is involved in gymnastics and has been since age 5. In addition to training for the gymnastics team, she runs 10 miles per day. She is proud of her athleticism and states that she is still too fat and wants to lose more weight. You note that her BMI is 15. She denies eating excessively or making herself vomit. The patient's mother states she barely eats. This patient is at risk for which medical problem? (A) Obesity (B) Bulimia (C) Anorexia nervosa (D) Not at risk; normal adolescence

(C) Anorexia nervosa This patient has a preoccupation with her weight and feels that she is too fat. She has a BMI of 15, which is underweight, and does not see herself as too thin. The patient's mother states that she barely eats, which is typical of patients with anorexia. With a BMI of 15 and a daily exercise routine, the patient is hardly at risk for obesity. Although this patient is difficult to classify without further information, you believe her when she states that she doesn't eat excessively or make herself vomit. Patients with bulimia will go on binges of overeating, then make themselves vomit. Although normal adolescents are concerned with their body image, they do not exercise to the extent that this girl does nor do they typically have a BMI of 15 and barely eat.

A 63-year-old high school coach presents to your office complaining of weakness in both of his hands. He states that he has trouble writing out test questions and gripping the baseball bat in practice. He also complains that the muscles in his forearms are twitching. He has a 10-year history of hypertension, which is well controlled, and has no history of trauma. He has a 50-year history of chewing tobacco and drinks three to four beers two to three times per month. He denies any cigarette or drug use. On exam, his grip strength is greatly decreased bilaterally but his sensation is normal. His brachioradialis reflexes are also decreased. You note some wasting of the muscles between the thumbs and fingers on the dorsal surface of both hands. In what part of the peripheral nervous system is he most likely to have a disorder? (A) Spinal roots (B) Neuromuscular junction (C) Anterior horn cells (D) Muscle fibers

(C) Anterior horn cells In anterior horn cell disease, there is both weakness and atrophy in a focal or segmental pattern. Sensation remains intact, while reflexes are decreased. When seen in childhood, the problem is often polio; in older adults, it is often amyotrophic lateral sclerosis (Lou Gehrig's disease). Fasciculations in the muscle fibers are often felt and seen.

An 85-year-old retired secretary presents to your office complaining of severe lower back pain. She states that it started 4-5 months ago and has gradually been getting worse. She takes Daypro for arthritis but that hasn't helped her very much. She says that lying down or sleeping does not seem to help. She doesn't recall any trauma to her back. She has had no unusual problems with strength in her legs or bowel or bladder control. Her past medical history is significant for 30 years of hypertension and right-sided breast cancer 10 years ago. She had a mastectomy and radiation therapy. She did not take any chemotherapy. She denies tobacco or alcohol use. On exam, you find a thin, cachectic-appearing woman in no acute distress. Her heart and lung exam are normal. She is tender over the L4 and L5 vertebrae. She has normal sensation, strength, and reflexes in her lower extremities. She is nontender over the para spinal muscles around the lumbar region and has a negative straight leg rise bilaterally. (A) Mechanical low back pain (B) Radicular back pain (C) Back pain from metastatic disease (D) Spinal stenosis

(C) Back pain from metastatic disease Back pain from metastatic disease occurs even when the patient is at rest. It generally does not radiate, and the patient has normal reflexes, strength, and sensation. Often there is point tenderness where the metastatic disease is located. The pain in this patient most likely represents breast cancer metastasis.

A 65-year-old gardener presents to the office for difficulty with urination. Increasingly, he has noticed feeling like his bladder has not emptied completely and his urinary stream has a decreased force. He experiences dribbling after he completes urination. These symptoms have been going on for several years, but his level of discomfort has increased over the past few months. On physical examination, you feel a symmetrically enlarged, smooth, and firm prostate gland, which seems to protrude more into the rectal lumen without any discrete lesions. What is your most likely diagnosis? (A) Normal prostate (B) Prostatitis (C) Benign prostatic hyperplasia (D) Prostate cancer

(C) Benign prostatic hyperplasia The patient's symptoms and physical exam are most consistent with benign prostatic hyperplasia. The affected prostate gland is symmetrically enlarged, smooth and firm, although slightly elastic. It may protrude more into the rectal lumen. The median sulcus may be obliterated as well. Further diagnostic tests may be required to rule out prostate cancer.

A 6-month-old infant presents with her parents for a routine checkup. You obtain a history and ask questions to obtain information on developmental stage. The answers given by the parents indicate that the infant is significantly behind in her development. The infant is in the 50th percentile for weight, height, and head circumference. Which of the following conditions is a potential cause of developmental delay? (A) Otitis media (B) Overfeeding (C) Gastroesophageal reflux (D) Neonatal meningitis

(D) Neonatal meningitis Neonatal meningitis is a significant infection, which can result in significant developmental delay.

A 48-year-old homemaker comes to your office complaining of a breast lump she found under her right arm while showering. She says it hasn't been there during past checks and it doesn't hurt when she examined it. She admits she hadn't done a self-breast exam in almost 1 year. She states she has had three normal mammograms during the last 8 years. The last mammogram was 18 months ago. Her past medical history is significant for one spontaneous vaginal delivery at age 38. She went through menopause 3 years ago and is on estrogen and progesterone replacement. She has smoked one pack of cigarettes daily for the past 25 years but denies any alcohol or drug abuse. Past medical history reveals that her maternal aunt had breast cancer in her thirties and her maternal grandmother had ovarian cancer in her seventies. Review of systems is noncontributory. On exam, you feel a hard, approximately 2-cm lump in the tail of Spence under the patient's right arm. The lump is nontender, and you have difficulty finding the borders. It is not adherent to anything, and you feel no lymphadenopathy in the axilla or supraclavicular areas. The skin over the breast appears normal. Examination of the left breast is unremarkable. What type of breast mass is the patient most likely to have? (A) Fibroadenoma (B) Cysts (C) Cancer

(C) Cancer Breast cancer is very rarely seen under the age of 30 and usually strikes middle-aged and elderly women. It can start either before or after menopause. There is usually a single lump. The masses are usually irregular in shape, hard, and non-tender. They can be fixed to underlying tissue. In advanced cases, skin retractions are noted.

A 68-year-old retired mechanic comes to the office for an annual checkup. He is concerned that the small red areas on his skin may be cancer. On physical examination, you see round, bright, ruby-red lesions that are 1-3 mm in diameter and scattered on the anterior chest; they partially blanch with pressure. What is the most likely diagnosis? (A) Squamous cell carcinoma (B) Actinic keratosis (C) Cherry angioma (D) Spider angioma

(C) Cherry angioma Cherry angiomas are bright or ruby red, usually 1-3 mm in diameter, and round and flat, which may show partial blanching with pressure. They increase in size and number as a person ages and do not confer an increased risk of skin cancer.

Mr. T. is complaining of a headache. He has just awakened from a nap and states that the headache always begins behind one eye, "feels like an icepick," and is accompanied by tearing and a runny nose. You suspect Mr. T has: (A) Migraine headache (B) Tension headache (C) Cluster headache (D) A stroke

(C) Cluster headache A cluster headache produces pain around the eye, temple, forehead, and cheek; it is unilateral and always on the same side of the head. The pain may be excruciating and can occur as often as twice a day. This type of headache is more common in men than in women.

A 55-year-old construction worker presents to the clinic for evaluation of fatigue and weakness. During the course of the interview, the patient reveals that he has noticed both an increase in his ring size and his shoe size; you ask to look at his driver's license and also at any pictures that he has in his wallet of himself from an earlier time period. Upon looking at these pictures, you are able to strongly theorize that he has acromegaly. Which physical finding is most consistent with this diagnosis? (A) High forehead (B) Small, recessed jaw (C) Coarsening of the facial features (D) Ptosis

(C) Coarsening of the facial features

You have just completed your examination and you document that the patient is unable to differentiate between sharp and dull touch bilaterally. You interpret this as: (A) Bell's palsy (B) Scleroderma of the face and cheeks (C) Damage to cranial nerve V (trigeminal nerve) (D) Paralysis

(C) Damage to cranial nerve V (trigeminal nerve) Facial sensations of pain or touch are mediated by cranial nerve V. Facial paralysis can be central or peripheral. Bell's palsy results from a peripheral lesion of cranial nerve VII—the patient cannot wrinkle the forehead on the affected side nor make a smile, so both the upper and lower facial muscles are involved. In central lesions of cranial nerve VII from cerebrovascular accidents, the patient can wrinkle both sides of the forehead but cannot make a smile.

A 25-year-old graduate student presents to your clinic; she is concerned about her exposure to a communicable disease that occurred at a party she recently attended. You, the interviewer, enter the room and greet the patient and then ask what brought her into your office today. She states that she wants to establish care, so you start by asking her if she has any specific concerns that she would like you to address. She states that she is concerned that she may have gotten an STD. You continue to maintain eye contact and state, "Go on." Which type of interviewing technique have you demonstrated? (A) Echoing (B) Directed questioning (C) Facilitation (D) Reassurance

(C) Facilitation Facilitation is the type of interviewing technique that you have demonstrated. By maintaining eye contact and asking the patient to "go on," you have attempted to make her more comfortable in relating details about the concern that brought her into your office.

A 7-year-old boy is brought to the ER by EMS and is accompanied by his parents. They report that he had a spell an hour before, during which his body stiffened, he lost consciousness, and then started jerking. They also report that he bit his tongue. He was sitting quietly in the living room playing video games when this occurred. The parents state that the child has never had any problems like this in the past. Upon exam, he is pale and confused. He says he is sleepy and would like every one to go away. You notice blood in his mouth and that his shorts and underwear are soiled with urine. What is the mostly likely reason for this "spell?" (A) Simple partial seizure (B) Complex partial seizure (C) Generalized tonic clonic (grand mal) seizure (D) Absence seizure

(C) Generalized tonic clonic (grand mal) seizure Patients who experience a grand mal seizure often lose consciousness suddenly, with a stiffening of the body. This is often followed by the clonic jerking of the muscles. Tongue biting and urinary incontinence are often noted. The patient will then have a pronounced postictal state of confusion and drowsiness.

A 42-year-old female office worker presents to your clinic complaining of awakening with an exquisitely tender, red, swollen right knee. She has never had this occur before and has had no history of trauma or recent illness. She has no significant past medical history. A family history reveals that her father had similar problems in middle age. On exam, she has a red, warm, tender right knee with decreased range of motion. Tapping the fluid in the knee showed no signs of infection indicating a probable arthritic cause. The most likely form of arthritis would be: (A) Rheumatoid arthritis (B) Degenerative joint disease (C) Gouty arthritis (D) Polymyalgia rheumatica

(C) Gouty arthritis Gouty arthritis usually occurs spontaneously, after a night in bed or a large meal. It results in swollen, red, warm, very tender joints. It is caused by an increase in crystals within the joint.

You have just completed a vaginal examination of a woman who is 6 weeks pregnant. You read on her chart that her cervix is softened and looks cyanotic. You know that she is exhibiting which of the following signs? (A) Chadwick's sign and Hegar's sign (B) Goodell's sign and Chadwick's sign (C) Hegar's sign and Goodell's sign (D) Tanner's sign and Hegar's sign

(C) Hegar's sign and Goodell's sign Shortly after the first missed menstrual period, the female genitalia show signs of the hormonal changes of pregnancy. The cervix softens (Goodell's sign) at 4-6 weeks, and the vaginal mucosa and the isthmus of the uterus softens (Hegar's sign) at 6-8 weeks. The cervix looks cyanotic (Chadwick's sign) at 8-12 weeks. These changes reflect increased vascularity and edema of the cervix, and hypertrophy and hyperplasia of the cervical glands. Tanner's signs refer to sexual maturity ratings in adolescents.

A 60-year-old homemaker presents to your office complaining of swelling to her left lower leg. She states that it has been developing over the last few weeks since her surgery for ovarian cancer. She has a past medical history significant for four spontaneous vaginal deliveries and the recent diagnosis of ovarian cancer. She denies any drug, tobacco, or alcohol use. On review of systems, she denies any chest pain, palpitations, or shortness of breath. On exam, you note that not only are her leg and ankle swollen but so are her toes. The skin appears to be thicker than usual, but there are no signs of ulceration. What cause of peripheral edema is the most likely in her case? (A) Pitting edema (B) Chronic venous insufficiency (C) Lymphedema

(C) Lymphedema Lymphedema is swelling in the extremities caused by blockage in lymphatic return. This obstruction is often due to tumor, trauma, inflammation; it is rarely infectious. The swelling includes the toes and often causes thickening of the skin. Ulcerations are rare. In this case, the damage to the lymph ducts occurred during surgery.

A 6-month-old is brought to your clinic by his mother for a routine checkup. She is concerned, because he has large bluish marks on his buttocks and back. She is worried that people will think that she is harming him somehow. Based on your physical examination, he is completely healthy and his development is on track for his age. What is your most likely diagnosis of this skin rash? (A) Café au lait spots (B) Salmon patch/stork bite (C) Mongolian spots (D) Neurofibromatosis

(C) Mongolian spots Mongolian spots are more common among darker-skinned babies. They range from bluish-gray to deep brown or black. They disappear after a few years and usually fade by puberty.

A 40-year-old flight attendant presents to your office for evaluation of abdominal pain. It is worse after eating, especially if she has a meal that is spicy or high in fat. She has tried over-the-counter antacids, but they have not helped the pain. After examining her abdomen, you strongly suspect cholecystitis. Which sign on examination increases your suspicion for this diagnosis? (A) Psoas sign (B) Rovsing's sign (C) Murphy's sign (D) Grey Turner's sign

(C) Murphy's sign

During your examination you note that your client's eyes are puffy. You further assess coarse and dry hair. You suspect: (A) Scleroderma (B) Cachexia (C) Myxedema (D) Cretinism

(C) Myxedema Myxedema arises from hypothyroidism, or a deficiency of thyroid hormone. You may see a periorbital edema, coarse facial features, dry skin, and dry coarse hair and eyebrows. Sclerodema is a rare autoimmune disorder affecting blood vessels and connective tissue, causing a tightening of the skin in the lower face. Cachexia refers to severe weight loss from poor nutrition, which may result in coarse hair but not puffiness. Cretinism arises from severe congenital hypothyroidism and is associated with dwarfism and mental retardation.

A 68-year-old house painter comes to your office complaining of an ulcer on his right foot. He just noticed it yesterday, although he thinks it is quite large. He finds this strange because it doesn't hurt at all. He wears heavy work boots and does a good deal of climbing on ladders. He doesn't recall stepping on any nails. His past medical history is significant for 15 years of type-2 diabetes. He states the first 5 years following his diagnosis, he didn't take the medicine as he should have and that he is now on insulin shots. He also has 10 years of high blood pressure. He has smoked two packs of cigarettes for the last 50 years, but he denies recent alcohol or drug use. His family history is significant for a mother and aunt with high blood pressure and diabetes. On exam, you see a 2-cm ulcer over the plantar surface of the foot proximal to the great toe. There is callus formation around the edge of the ulcer. The patient is unable to distinguish vibration from pressure on his great toe. There are no color changes to his foot between elevation and dependency. What type of common ulcer best describes his problem? (A) Arterial insufficiency ulcer (B) Venous insufficiency ulcer (C) Naturopathic ulcer

(C) Neuropathic ulcer Neuropathic ulcers appear at areas of pressure points because the person affected is unable to perceive that there is pain in the area. The peripheral neuropathy accounts for the lack of pain and the occasional late diagnosis.

A 22-year-old woman comes to your office complaining of a white discharge from her breasts, which has been occurring for 2 months. She also states that she hasn't had her period in 6 months. She denies any chance of pregnancy since she hasn't been sexually active in over 1 year. She is on no medications except a multivitamin. Her past medical history is significant for allergies only. Past medical history reveals a mother with cystic breasts and a great aunt with breast cancer. Review of systems is noncontributory. On exam, her breasts are symmetrical, with no skin changes. You are able to express milky discharge from each nipple. You feel no discrete masses, and her axillae are normal. The remainder or her heart, lung, abdominal, and pelvic exam are unremarkable. What cause of nipple discharge is the most likely in her circumstance? (A) Benign breast abnormality (B) Breast cancer (C) Nonpuerperal galactorrhea

(C) Nonpuerperal galactorrhea Galactorrhea, or milky discharge, when not due to pregnancy or lactation is called nonpuerperal. It is usually has hormonal or pharmacologic associations. In this case, a prolactinoma in the pituitary gland would be at the top of the differential, given the galactorrhea and the amenorrhea. Nipple discharge in benign breast abnormalities tends to be clear and unilateral. Nipple discharge in breast cancer is usually unilateral and can be clear or bloody.

A 42-year-old retail manager is sitting in your clinic exam room, waiting for a routine checkup for hypertension. You enter the room and sit down, making eye contact, and move your body slightly toward the patient. You nod your head as the patient begins to bring you up to date on his life since his last appointment. This interviewing technique is an example of: (A) Echoing (B) Adaptive questioning (C) Nonverbal communication (D) Empathic responses

(C) Nonverbal communication You have used nonverbal communication through the techniques of eye contact, sitting down, moving your body in slightly toward the patient, and nodding. Echoing is not the correct response since it involves simple repetition of the patient's words, and, as the interviewer in this scenario, you haven't repeated the patient's words. Adaptive questioning is also incorrect. It involves using various questions to clarify the patient's story, and in this scenario, as the interviewer, you haven't asked any questions yet. Empathic response is not correct. You have not identified the patient's feelings; and she has not expressed any during the interview.

A 76-year-old widow comes to your clinic to establish care. She admits that she hasn't seen a clinician in over 5 years, but her daughter has been nagging her to be seen. She states she has noticed that an area to the outside of her left breast feels enlarged, and there are some different changes to the skin in the area. Her past medical history is significant for two spontaneous vaginal deliveries and diet-controlled diabetes. She denies any tobacco or drug use and no alcohol abuse. Her review of systems is remarkable for a 10-pound weight loss in the last 6 months and some recent fatigue. On exam, you find an elderly woman appearing her stated age. Inspection of her left breast reveals some thickening of the skin lateral to her areola, with enlarged pores. You feel a non-tender 5-cm mass. The axilla has at least two enlarged lymph nodes, which are adherent to the underlying ribs. The right breast and axilla are unremarkable. What visible skin change of the breast does she have? (A) Nipple retraction (B) Paget's disease (C) Peau d'orange sign

(C) Peau d'orange sign Peau d'orange, or orange-peel sign, is caused by local lymphatic blockage due to tumor. The involved skin then becomes thickened, and the pores become more prominent. This is usually a late sign of breast cancer. Choice (A) is incorrect. A retracted nipple is flattened or pulled inward. The surrounding skin is not thickened, nor are there enlarged pores. Paget's disease is also incorrect. This uncommon form of breast cancer starts as an eczema-like scaly skin change around the areola. The lesion may weep, crust, or erode.

A 38-year-old landscape designer presents to the clinic to be evaluated for a cough. His vital signs show a temperature of 103ºF, a pulse of 120, and a blood pressure of 88/55. His respiratory rate is 28. He appears to have labored breathing. All of these indicators are signs of: (A) Pain from the cough (B) Anxiety about the visit (C) Respiratory distress

(C) Respiratory distress The patient has classic signs of respiratory distress. He is tachypneic — his respiratory rate is greater than 20 breaths/minute and he appears to have labored breathing. Normal oral temperature is between 96.4º F and 99.1ºF, normal pulse is between 60 and 100 beats per minute, normal blood pressure is a systolic of 90/60 to 140/90, and a normal respiratory rate is between 14 and 20 breaths per minute.

A 15-year-old high school student presents to the clinic with a 1-day history of nausea and anorexia. He describes the pain as generalized yesterday, but today it has localized to the right lower quadrant. You palpate the left lower quadrant and the patient experiences pain in the right lower quadrant. What is the name of this sign? (A) Psoas sign (B) Obturator sign (C) Rovsing's sign (D) Cutaneous hyperesthesia

(C) Rovsing's sign

A 58-year-old college professor presents to your clinic to establish care. In taking vital signs, you feel the patient's radial pulse. The pulse pressure is diminished, and the pulse feels weak and small. The upstroke feels a little slow, and the peak is prolonged. You immediately suspect that this patient may have which of the following diagnoses? (A) Hypertrophic cardiomyopathy (B) Premature atrial contractions (C) Severe aortic stenosis (D) Constrictive pericarditis

(C) Severe aortic stenosis Severe aortic stenosis is one condition in which the pulse pressure is diminished and the pulse feels weak and small. Other conditions that could result in this same type of pulse include heart failure, hypovolemia, exposure to cold, and severe congestive heart failure.

Which of the findings is a normal finding when examining the glands? (A) The skin is wrinkled and without lesions (B) The dorsal vein may be visible (C) Smegma may be present under the foreskin of an uncircumcised male (D) There is no hair

(C) Smegma may be present under the foreskin of an uncircumcised male Some cheesy smegma may collect under the foreskin of an uncircumcised male. The glans looks smooth and without lesions. Pubic lice and nits settle in the base of the penis and excoriations or wrinkling may suggest activity. Hair distribution is variable.

A 24-year-old male graduate student is brought into the ER following a two-vehicle MVA. He is on a backboard and has obvious bruises and lacerations to his head and extremities. On exam, he does not respond to either conversation or even shaking. He does arouse to a deep sternal chest rub and mumbles incoherently. When the stimulus ceases, he lapses back into an unresponsive state. What level of consciousness best describes his response? (A) Lethargic (B) Obtunded (C) Stuporous (D) Comatose

(C) Stuporous A stuporous patient only responds to painful stimuli such as a sternal rub or the pinching of a tendon. When the painful stimulus is removed, he becomes unresponsive again. A lethargic patient awakens to conversation, answers questions, and then returns to sleep. An obtunded patient often must be shaken to respond. He often looks confused and gives slow answers. A comatose patient will not respond to any external stimuli and will remain in an unarousable state.

In performing the bimanual examination, you note that the cervix feels smooth and firm, is round, and is fixed. Your cervical palpation produces some pain. The best interpretation of these results is: (A) These findings are well within normal limits (B) It is unusual to have pain when palpating the cervix, but the rest of the findings are within normal limits (C) The cervix should move when palpated; an immobile cervix may indicate malignancy (D) The cervical consistency should be soft and velvety and not firm

(C) The cervix should move when palpated; an immobile cervix may indicate malignancy. Characteristics of a normal cervix are: consistency is smooth and firm; contour is evenly rounded; and mobility is flexible. An abnormal cervix would be firm, fixed, possibly nodular, and painful when moved.

When observing the vestibule, you should be able to see which of the following anatomic structures? (A) Urethral meatus and paraurethral (Skene's) glands (B) Vaginal orifice and vestibular (Bartholin's) glands (C) Urethral meatus and vaginal orifice (D) Paraurethral (Skene's) and vestibular (Bartholin's) glands

(C) Urethral meatus and vaginal orifice The labial structures encircle a boat-shaped space called the vestibule. Within it are several openings. The urethral meatus appears as a small, raised structure between the clitoris and the vagina. Just posterior to the urethral meatus at the 5 and 7 o'clock positions are the tiny openings of the paraurethral (Skene's) glands. The vaginal orifice is posterior to the urethral meatus. On either side and posterior to the vaginal orifice are two vestibular (Bartholin's) glands, which secrete a clear lubricating mucus during intercourse. Their ducts are not visible but open in the groove between the labia minora and the hymen.

When instructing patients about how to perform a testicular self-exam, which of the following statements is most correct: (A) "A good time to examine your testicles is just before you take a shower." (B) "Perform the testicular exam at least once a week to detect the early stages of cancer." (C) "The testicle should feel firm and has a lumpy consistency, be movable, and shaped like an egg." (D) "If you notice any change from what you normally see or feel, call your health care provider."

(D) "If you notice any change from what you normally see or feel, call your health care provider." A firm painless lump, a hard area, or an enlarged testicle is each an abnormal and unexpected finding, which should be assessed by a trained clinician. Time of week and time of day are not factors men should consider relating to self-testicular examination. Regular examination should be encouraged, and changes or abnormal findings need to be reported.

Mrs. W is in her first trimester of pregnancy. She is experiencing significant nausea and vomiting and asks if it will get better. Your response is: (A) "At about the time the baby moves, it will get better.'' (B) "Many women experience nausea and vomiting until the third trimester." (C) "Did your mother have significant nausea and vomiting?" (D) "Usually, by the beginning of the second trimester, the nausea and vomiting improve."

(D) "Usually, by the beginning of the second trimester, the nausea and vomiting improve." The nausea, vomiting, and the fatigue of pregnancy usually improve by weeks 12-16. Fetal movement occurs at approximately 20 weeks gestation.

A 32-year-old farmer presents to the clinic for evaluation of pain with defecation as well as rectal bleeding. His family history is negative for inflammatory diseases of the bowel and for colon cancer. On physical examination, there is a swollen skin tag, and, with gentle separation of the anal margin, a thin line of ulceration is visible. What is your most likely diagnosis? (A) Pilonidal cyst (B) Anorectal fistula (C) External thrombosed hemorrhoid (D) Anal fissure

(D) Anal fissure An anal fissure is a painful oval ulceration of the anal canal and is found most commonly in the midline posteriorly inside the rectum/anal canal. Inspection of the anus may show a swollen "sentinel" skin tag just below it, and gentle separation of the anal margins may reveal the lower edge of the fissure.

A 55-year-old realtor presents to your office for evaluation of a skin lesion that has been present for a long time. She came in because her daughter had been reading about skin cancer in her favorite women's magazine and was worried about her mother. The lesion is located on her right cheek. There is a central depression and a firm elevated border. Initially, when it first appeared 5 years ago, it was translucent. What is the most likely diagnosis? (A) Actinic keratosis (B) Seborrheic keratosis (C) Squamous cell carcinoma (D) Basal cell carcinoma

(D) Basal cell carcinoma A basal cell carcinoma usually appears on the face. Initially, it is a translucent nodule, which spreads, leaving a depressed center and a firm elevated border. Although it is a malignant tumor, it grows slowly and rarely metastasizes.

A 42-year-old homemaker presents to your office complaining of swelling and pain in her right lower leg. She states it developed yesterday upon returning from a driving vacation to the Grand Canyon. She also informs you that she hiked and climbed all through the canyon with no pain in her legs or shortness of breath. Her past medical history is significant for one spontaneous vaginal delivery and one caesarean section for fetal distress. She is on a daily multi-vitamin and a low-estrogen birth control pill. She drinks socially one to two times a month and denies any tobacco or drug use. Her father has type-2 diabetes since his early 50s. On exam, you see that her right lower leg does look larger than the left. You measure 15 cm above the heel on each leg and find that the right leg is 3 cm larger than the left. There is no redness or heat over the calf. Placing your thumb over the tibia shows pitting edema. Palpating the back of her calf is negative for cords. What is the peripheral vascular disorder that best describes her problem? (A) Acute cellulitis (B) Raynaud's disease (C) Atherosclerosis obliterans (D) Deep venous thrombosis

(D) Deep venous thrombosis Deep venous thrombosis is from clot formation in the deep draining veins of the calves and thighs. It can cause an aching pain and swelling of the extremity. Cords, which are associated with more superficial thrombosis, are generally not felt.

An 83-year-old woman presents to the ER the afternoon of Thanksgiving complaining of severe abdominal pain. She states it was at first cramping around her navel but has now radiated to the rest of her abdomen. She also complains of some vomiting and bloody diarrhea. About 1 hour before the pain started, she had a large traditional holiday meal. She states she generally only eats about half a sandwich and some soup at meals. She denies any earlier history of similar pain. Her past medical history is significant for coronary artery disease, for which she has had a stent placed in the past. She is only taking medicines related to her heart condition. She denies any alcohol, drug, or tobacco use. Her review of systems is positive for soreness in her legs while walking and occasional chest pain on exertion. On exam, you find an elderly woman in great distress. She is pale and tachycardic. Examining her abdomen, you hear decreased bowel sounds. On palpitation, her abdomen is rigid with voluntary guarding and rebound. Her rectal exam shows grossly bloody stool. What diagnosis for abdominal pain best describes her symptoms and signs? (A) Acute pancreatitis (B) Acute cholecystitis (C) Acute appendicitis (D) Mesenteric ischemia

(D) Mesenteric ischemia Mesenteric ischemia is caused from thrombosis, emboli, or hypoperfusion. It often begins periumbilically, radiating diffusely. Severe pain, vomiting, and bloody diarrhea are common. In this patient it is not surprising, in light of known cardiac and peripheral vascular disease, that a large meal would have precipitated such a condition.

A 26-year-old woman follows up at your office for her 6-week postpartum visit. She complains of severe constipation problems since the birth of her last child. Although she says she is eating healthy and drinking plenty of fluids, the problem persists. She does complain of pain with defecation. She denies any abdominal pain, nausea, vomiting, or diarrhea. She has never had problems with constipation except for during her pregnancies. Her past medical history is significant for two vaginal deliveries. She denies any tobacco, drug, or alcohol use. She is currently taking only prenatal vitamins. She has no family history of any bowel problems. On exam, she has normal bowel sounds, and her abdomen is soft with no rebound or guarding. Her pelvic exam reveals that her uterus has returned to nongravid size and that her episiotomy site has healed. Her rectal exam is painful and is positive for occult blood. What is the best choice for the cause of her constipation? (A) Obstructing lesion (B) Irritable bowel syndrome (C) Rectal cancer (D) Painful anal lesions

(D) Painful anal lesions Any lesion around the anus, including hemorrhoids (common in pregnancy), or the repair of a perineal birth injury can cause pain severe enough that it can lead to constipation. In this case, the positive occult blood was from enlarged painful internal hemorrhoids.

A 15-year-old boy comes to your clinic complaining about pain for the past 2 weeks on the bottom of his left foot. He states that he stepped on a sticker outside and ever since he has had a spot there that hurts. He has no significant past medical history. On exam, you note an 8-mm area of thickened skin just proximal to the great toe on his left foot. There is no redness, warmth, or drainage from the area. There are, however, small dark dots present underneath the thickened skin. Pressing against the area causes the patient significant pain. What disorder of the foot is most likely the diagnosis for his case? (A) Corn (B) Callus (C) Neuropathic ulcer (D) Plantar wart

(D) Plantar wart Plantar warts often occur on the metatarsal area, heels, and toes, where there has been slight trauma in the past. They cause thickened skin and can be very painful upon pressure. There are usually characteristic black dots associated with the wart, which are thrombosed capillaries. Corns are thickened skin, but they are caused by pressure against the foot. They occur most commonly on the lateral side of the 5th toe and between the 4th and 5th toes. Callus formations also occur at pressure points but at places where the skin is normally thick. Calluses are not painful and are usually located around the heel or over the plantar surface, just proximal to the great toe. They are not tender. Neuropathic ulcers also develop at pressure points, usually because of a lack of sensation. Although often deep and infected, they are not painful. They are usually red, warm, and sometimes drain serosanguinous fluid.

A 54-year-old woman who works at a dry cleaners presents to your office complaining that she cannot do overhead work with her right arm. She states that she can no longer lift her arm over her head without using her other arm to prop it up. She remembers no specific injury but has been doing overhead work at the dry cleaners for over 15 years. Her past medical history is significant for three spontaneous vaginal deliveries and 15 years of hypertension. On exam, she has demonstrative weakness with internal and external rotation on the right. When she attempts to abduct her arm, there is obvious shrugging of the shoulder. On inspection of her back, you note atrophy of the muscles surrounding the shoulder and scapula. She is nontender over any boney prominence in the shoulder. Her passive range of motion is normal, while on active range of motion she is unable to elevate her arm more than 12 inches and cannot raise it to shoulder level. What shoulder problem is the most likely diagnosis? (A) Rotator cuff tendonitis (B) Bicipital tendonitis (C) Acrominoclavicular arthritis (D) Rotator cuff tear

(D) Rotator cuff tear Rotator cuff tear is often seen in people over 40 due to a sudden trauma or repeated impingement weakening the rotator cuff group. This often eventually leads to a complete tear. Weakness is common, as is atrophy of the muscles. With a complete tear, active abduction and flexion causes shoulder shrugging.

A 20-year-old college junior is brought to your office by classmates who state that she is acting strangely. She has been seen talking to herself around campus and has told people that she has been awarded the Nobel Prize for her study in physics. Her friends tell you that until 2 months ago she seemed a normal, happy young woman with no medical problems. They had heard mention of an older brother with mental problems. They tell you she drinks alcohol on the weekends but has never gotten drunk and has never done any drugs. On exam, you find you cannot understand her because she speaks in a nonsensible manner by repeating phrases and shifting her ideas. She is oriented to person, place, and time; her math calculations are correct, but she cannot explain any proverbs. You also notice that her blouse is on backwards. What is the most likely cause of her mental health problem? (A) Major depression (B) Bipolar disorder (C) Generalized anxiety disorder (D) Schizophrenia

(D) Schizophrenia Schizophrenia often starts suddenly in young adults with strong family history of mental health problems. Delusions, hallucinations, disorganized speech, disorganized behavior, and negative symptoms (flat affect, lack of interest or drive) are often present. In this case, the young woman has delusions (winning the Nobel Prize as a college student), hallucinations (apparent when she is seen having conversations with herself), disorganized speech (with shifting of ideas and clanging speech), and disorganized behavior (wearing clothes backwards).

Which of the following statements is true when performing a male genital examination? (A) Auscultate for the presence of bowel sounds over the scrotum in all males (B) Palpate for the vertical chain of lymph nodes along the groin inferior to the inguinal ligament (C) Palpate the inguinal canal only if there is a bulge present in the inguinal region during inspection (D) When palpating for a hernia on the right side, have the client shift his standing weight onto the left leg

(D) When palpating for a hernia on the right side, have the client shift his standing weight onto the left leg Positioning to the left leg allows the examiner to follow the inguinal canal and document any hernia-related bulge.

Which of the following statements regarding reinforcement when assessing reflexes is true? A) Used when reflexes are symmetrically hyperactive B) Technique involves isometric contraction of other muscles C) Supports the unsteady patient D) All of the above

B) Technique involves isometric contraction of other muscles Used when reflexes are symmetrically diminished or absent

Mrs. Hill is a 28-year-old African-American with a history of SLE (systemic lupus erythematosus). She has noticed a raised, dark red rash on her legs. When you press on the rash, it doesn't blanch. What would you tell her regarding her rash? A) It is likely to be related to her lupus. B) It is likely to be related to an exposure to a chemical. C) It is likely to be related to an allergic reaction. D) It should not cause any problems.

A) It is likely to be related to her lupus. A "palpable purpura" is usually associated with a vasculitis. This is an inflammatory condition of the blood vessels often associated with systemic rheumatic disease. It can cut off circulation to any portion of the body and can mimic many other diseases in this manner. While allergic and chemical exposures may be a possible cause of the rash, this patient's SLE should make you consider vasculitis.

An 8-year-old girl comes with her mother for evaluation of hair loss. She denies pulling or twisting her hair, and her mother has not noted this behavior at all. She does not put her hair in braids. On physical examination, you note a clearly demarcated, round patch of hair loss without visible scaling or inflammation. There are no hair shafts visible. Based on this description, what is your most likely diagnosis? A) Alopecia areata B) Trichotillomania C) Tinea capitis D) Traction alopecia

A) Alopecia areata This is a typical description for alopecia areata. There are no risk factors for trichotillomania or for traction alopecia. The physical examination is not consistent with tinea capitis because the skin is intact.

Pain, swelling, loss of both active and passive motion, locking, and deformity would be consistent with which of the following? A) Articular joint pain B) Bursitis C) Muscular injury D) Nerve damage

A) Articular joint pain These features are consistent with articular joint pain, whereas the other problems are associated with extra-articular structures

A mother brings her 11 month old to you because her mother-in-law and others have told her that her baby is jaundiced. She is eating and growing well and performing the developmental milestones she should for her age. On examination you indeed notice a yellow tone to her skin from head to toe. Her sclerae are white. To which area should your next questions be related? A) Diet B) Family history of liver diseases C) Family history of blood diseases D) Ethnicity of the child

A) Diet The lack of jaundice in the sclerae is an important clue. Typically, this is the first place where one sees jaundice. This examination should also be carried out in natural light (sunlight) as opposed to fluorescent lighting, which can alter perceived colors. Many infants this age have a large proportion of carrots, tomatoes, and yellow squash, which are rich in carotene. Liver and blood diseases can cause jaundice, but this should involve the sclerae. The ethnicity of the child should not cause a perceived change from her usual skin tone.

An 85-year-old retired housewife comes with her daughter to establish care. Her daughter is concerned because her mother has started to fall more. As part of her physical examination, you ask her to walk across the examination room. Which of the following is not part of the stance phase of gait? A) Foot arched B) Heel strike C) Mid-stance D) Push-off

A) Foot arched The foot when it is flat is part of the stance phase of gait, not the foot when it is arched

A 38-year-old unemployed man presents to the ER complaining of black tarry stools that now have blood in them. He states he has also had some severe pain in his stomach for the last few days. He vomited once and thinks he saw some blood. His past medical history is significant for pancreatitis and asthma. He has smoked two packs of cigarettes for 25 years and drinks approximately 12 beers a day. He denies any IV drug use. He has had no recent out-of-state travel. On review of symptoms, he notes that he has fainted in the last day. On exam, you find a cachetic man appearing older than his stated age. He is afebrile but tachycardic, at 120 bpm. Otherwise, his heart and lung exam are normal. On inspection, he has no dilated veins around his umbilicus or skin of an unusual color. Increased bowel sounds are heard during auscultation. Palpation reveals diffuse tenderness, which is more severe in the epigastric area. His liver percusses 12 cm in the right midclavicular line, and he has no fluid wave. He is grossly positive for blood on his rectal exam. What cause of black stools most likely describes his symptoms and signs? (A) Gastritis (B) Inflammatory infectious diarrhea (C) Mallory-Weiss tear (D) Esophageal varices

A) Gastritis Severe gastritis is often caused by alcohol ingestion. Bleeding from acute gastritis will be black when the transit time from the colon is slow. When the transit time increases, frank blood can be present. Bowel sounds are often increased, and the abdomen can be diffusely tender.

Mrs. Fletcher complains of numbness of her right hand. On examination, sensation of the volar aspect of the web of the thumb and index finger and the pulp of the middle finger are normal. The pulp of the index finger has decreased sensation. Which of the following is affected? A) Median nerve B) Ulnar nerve C) Radial nerve

A) Median nerve The pulp of the index finger is innervated by the median nerve. A decrease in sensation at this area would support a diagnosis of carpal tunnel syndrome. The pulp of the fifth finger is supplied by the ulnar nerve, and the dorsal web space of the thumb and index finger is supplied by the radial nerve.

A new mother is concerned that her child occasionally "turns blue." On further questioning, she mentions that this is at her hands and feet. She does not remember the child's lips turning blue. She is otherwise eating and growing well. What would you do now? A) Reassure her that this is normal B) Obtain an echocardiogram to check for structural heart disease and consult cardiology C) Admit the child to the hospital for further observation D) Question the validity of her story

A) Reassure her that this is normal This is an example of peripheral cyanosis. This is a very common and benign condition which typically occurs when the child is slightly cold and his peripheral circulation is adjusting to keep his core warm. Without other problems, there is no need for further workup. If the lips or other central locations are involved, you must consider other etiologies.

A middle-aged man comes in because he has noticed multiple small, blood-red, raised lesions over his anterior chest and abdomen for the past several months. They are not painful and he has not noted any bleeding or bruising. He is concerned this may be consistent with a dangerous condition. What should you do? A) Reassure him that there is nothing to worry about. B) Do laboratory work to check for platelet problems. C) Obtain an extensive history regarding blood problems and bleeding disorders. D) Do a skin biopsy in the office.

A) Reassure him that there is nothing to worry about. These represent cherry angiomas, which are very common, benign lesions. Further workup such as laboratory work, skin biopsy, or even further questions are not necessary at this time. It would be wise to ask the patient to report any changes in any of his skin lesions, and tell him that you would need to see him at that time.

A 77-year-old retired school superintendent comes to the office with a report of unsteady hands. He says that for the past 6 months when his hands are resting in his lap they shake uncontrollably. He says that when he holds them out in front of his body or uses his hands, the shaking improves. He also complains of some difficulty getting up out of his chair and walking around. He denies any recent illnesses or injuries. His past medical history is significant for high blood pressure and coronary artery disease, requiring a stent in the past. He has been married for more than 50 years and has five children and 12 grandchildren. He denies any tobacco, alcohol, or drug use. His mother died of a stroke in her 70s and his father died of a heart attack in his 60s. He has a younger sister with arthritis. His children are all essentially healthy. Examination reveals a fine pill-rolling tremor of his left hand. His right shows less movement. His cranial nerve examination is normal. He has some difficulty rising from his chair, his gait is slow, and it takes him time to turn around to walk back towards the examiner. He has almost no "arm swing" with his gait. What type of tremor is most likely? A) Resting Tremor B) Postural Tremor C) Intention Tremor D) Athetoid Tremor

A) Resting Tremor

A 38-year-old woman comes to you and has multiple small joints involved with pain, swelling, and stiffness. Which of the following is the most likely explanation? A) Rheumatoid arthritis B) Septic arthritis C) Gout D) Trauma

A) Rheumatoid arthritis Rheumatoid arthritis is a systemic disease and accounts for multiple symmetrically involved joints. Septic arthritis is usually monoarticular, as are gout and trauma-related joint pain

A 68-year-old retired banker comes to your clinic for evaluation of left shoulder pain. He swims for 30 minutes daily, early in the morning. He notes a sharp, catching pain and a sensation of something grating when he tries overhead movements of his arm. On physical examination, you note tenderness just below the tip of the acromion in the area of the tendon insertions. The drop arm test is negative, and there is no limitation with shoulder shrug. The patient is not holding his arm close to his side, and there is no tenderness to palpation in the bicipital groove when the arm is at the patient's side, flexed to 90 degrees, and then supinated against resistance. Based on this description, what is the most likely cause of his shoulder pain? A) Rotator cuff tendinitis B) Rotator cuff tear C) Calcific tendinitis D) Bicipital tendinitis

A) Rotator cuff tendinitis Rotator cuff tendinitis is typically precipitated by repetitive motions, such as occurs with throwing or swimming. Crepitus/grating is noted in the shoulder with range of motion

You are a student in the vascular surgery clinic. You are asked to perform a physical examination on a patient with known peripheral vascular disease in the legs. Which of the following aspects is important to note when you perform your examination? A) Size, symmetry, and skin color B) Muscle bulk and tone C) Nodules in joints D) Lower extremity strength

A) Size, symmetry, and skin color

You are assessing a patient with joint pain and are trying to decide whether it is inflammatory or noninflammatory in nature. Which one of the following symptoms is consistent with an inflammatory process? A) Tenderness B) Cool temperature C) Ecchymosis D) Nodules

A) Tenderness Tenderness implies an inflammatory process along with increased temperature and tenderness.

What is a dermatome? A) a band of skin innervated by the sensory root of a single spinal nerve B) the gray matter of the spinal cord C) a nerve tumor in the skin D) none of the above

A) a band of skin innervated by the sensory root of a single spinal nerve

A 70-year-old patient with a history of hypertension has a blood pressure of 180/100 mm Hg and a heart rate of 90 beats per minute. The nurse hears an extra heart sound at the apex immediately before S1. The sound is heard only with the bell while the patient is in the left lateral position. With these findings and the patient's history, the nurse knows that the extra heart sound is most likely a(n): A) atrial gallop B) split S2 C) summation sound D) diastolic mumur

A) atrial gallop

The findings from an assessment of a 70-year-old patient with swelling in his ankles include jugular venous pulsations 5 cm above the sternal angle when the head of his bed is elevated 45 degrees. The nurse knows that this finding indicates: A) elevated pressure related to heart failure B) decreased fluid volume C) increased cardiac output D) narrowing of jugular veins

A) elevated pressure related to heart failure

A client with hearing loss by whisper test is further examined with a tuning fork, using the Weber and Rinne maneuvres. The abnormal results are as follows: bone conduction is greater than air on the left, and the client hears the tuning fork better on the left. Which of the following is most likely? A) otosclerosis of the L ear B) exposure to chronic loud noise of the L ear C) exposure to chronic loud noise of the R ear D) perforation of the R eardrum

A) otosclerosis of the L ear The above pattern is consistent with a conductive loss on the left side. Causes would include foreign body, otitis media, perforation, and otosclerosis of the involved side.

A 17-year-old high school senior presents to your clinic in acute respiratory distress. Between shallow breaths he states he was at home finishing his homework when he suddenly began having right-sided chest pain and severe shortness of breath. He denies any recent traumas or illnesses. His past medical history is unremarkable. He doesn't smoke but drinks several beers on the weekend. He has tried marijuana several times but denies any other illegal drugs. He is an honors student and is on the basketball team. His parents are both in good health. He denies any recent weight gain, weight loss, fever, or night sweats. On examination you see a tall, thin young man in obvious distress. He is diaphoretic and is breathing at a rate of 35 breaths per minute. On auscultation you hear no breath sounds on the right side of his superior chest wall. On percussion he is hyperresonant over the right upper lobe. With palpation he has absent fremitus over the right upper lobe.What disorder of the thorax or lung best describes his symptoms? A) spontaneous pneumothorax B) chronic obstructive pulmonary disease (COPD) C) asthma D) pneumonia

A) spontaneous pneumothorax Spontaneous pneumothorax occurs suddenly, causing severe dyspnea and chest pain on the affected side. It is more common in thin young males. On auscultation of the affected side there will be no breath sounds and on percussion there is hyperresonance or tympany. There will be an absence of fremitus to palpation. Given this young man's habitus and pneumothorax, you may consider looking for features of Marfan's syndrome. Read more about this condition.

What is the first sign of puberty in boys? A) testicle enlargement B) growth spurt C) penile enlargement D) appearance of pubic hair

A) testicle enlargement Puberty begins sometime between ages 9 and 13 1/2 years. The first sign is enlargement of the testes. Next, pubic hair appears and then penis size increases.

During percussion of the lungs, the nurse hears dullness on the left sternal border between the 3rd and 5th intercostal space. What is the most likely explanation? A) the heart is being percussed B) the patient has pneumonia C) the patient has a large lung mass D) none of the above

A) the heart is being percussed

Hypospadius is: A) urethral meatus opens along the ventral side of the penis B) testicle(s) have not descended into the scrotum C) there is no sperm in the testicle D) low testosterone

A) urethral meatus opens along the ventral side of the penis

A 15 year old female patient presents with a chief complaint of dizziness for 24 hours. You ask the patient to describe her dizziness and she says that the room is spinning. The clinical term that describes her symptom is: A) vertigo B) presyncope C) syncope D) disequilibrium

A) vertigo

During a speculum inspection of the vagina, the nurse would expect to see what at the end of the vaginal canal? A)Cervix B)Uterus C)Ovaries D)Fallopian tubes

A)Cervix At the end of the canal, the uterine cervix projects into the vagina

The following information is best placed in which category: "The patient was treated for an asthma exacerbation in the hospital last year; the patient has never been intubated." A. Adult illnesses B. Surgeries C. Obstetrics/gynecology D. Psychiatric

A. Adult illnesses This is information about a significant hospitalization and should be placed in the adult illnesses section. If the patient is being seen for an asthma exacerbation, you may consider placing this information in the present illness section, because it relates to the chief complaint at that visit.

Mark is a contractor who recently injured his back. He was told he had a "bulging disc" to account for the burning pain down his right leg and slight foot drop. The vertebral bodies of the spine involve which type of joint? A) Synovial B) Cartilaginous C) Fibrous D) Synostosis

B) Cartilaginous The vertebral bodies of the spine are connected by cartilaginous joints involving the discs. The elbow would be an example of a synovial joint, and the sutures of the skull are an example of a fibrous joint.

A 72-year-old teacher comes to your clinic for an annual examination. She is concerned about her risk for peripheral vascular disease and states that there is a place in town that does tests to let her know her if she has this or not. Which of the following disease processes is a risk factor for peripheral vascular disease? A) Gastroesophageal reflux disease B) Coronary artery disease C) Migraine headaches D) Osteoarthritis

B) Coronary artery disease Evidence of coronary artery disease implies that there is most likely disease in other vessels; therefore, this is a risk factor for peripheral vascular disease. Conversely, the presence of peripheral vascular disease is also a risk factor for coronary artery disease, and if present, it should be considered in reduction of cardiac risk factors.

A 29-year-old physical therapist presents for evaluation of an eyelid problem. On observation, the right eyeball appears to be protruding forward. Based on this description, what is the most likely diagnosis? A) Ptosis B) Exophthalmos C) Ectropion D) Epicanthus

B) Exophthalmos Exophthalmos is the condition when the eyeball protrudes forward. If it is bilateral, it suggests the presence of Graves' disease. If it is unilateral, it could still be caused by Graves' disease. Alternatively, it could be caused by a tumor or inflammation in the orbit.

Ray works a physical job and notes pain when he attempts to lift his arm over his head. When you move the shoulder passively, he has full range of motion without pain and there is no gross swelling or tenderness. What type of joint disease does this most likely represent? A) Articular B) Extra-articular C) Neither D) Both

B) Extra-articular This description fits extra-articular disease. Articular disease typically involves swelling and tenderness of the entire joint and limits both active and passive range of motion. This is most likely extra-articular because it affects a certain portion of the range of motion, is not painful with passive range of motion, and is not associated with gross swelling or tenderness

Which of the following might be a reason for unusually small testes size? A) cryptorchidism B) Klinefelter's syndrome C) orchitis D) testicular cancer

B) Klinefelter's syndrome Klinefelter's syndrome is a concern if very small testicles are detected.

Which of these findings would the nurse expects to notice during a cardiac assessment on a 4-year-old child? A) S3 when sitting up B) Murmur at second left intercostal space when supine C) Persistent tachycardia above 150 D) Palpable apical pulse in 5th intercostal space lateral to midclavicular line

B) Murmur at second left intercostal space when supine Some murmurs are common in healthy children or adolescents and are termed innocent or functional. The innocent murmur is heard at the second or third L intercostal space and disappears with sitting, and the young person has no associated signs of cardiac dysfunction.

Louise, a 60-year-old, complains of left knee pain associated with tenderness throughout, redness, and warmth over the joint. Which of the following is LEAST helpful in determining if a joint problem is inflammatory? A) Tenderness B) Pain C) Warmth D) Redness

B) Pain Pain is present in both inflammatory and noninflammatory conditions. Warmth, redness, and tenderness to palpation should lead one to consider an inflammatory etiology for the pain.

A high school football player injured his wrist in a game. He is tender between the two tendons at the base of the thumb. Which of the following should be considered? A) DeQuervain's tenosynovitis B) Scaphoid fracture C) Wrist sprain D) Rheumatoid arthritis

B) Scaphoid fracture The "anatomic snuffbox" is found between the extensor and abductor tendons at the base of the thumb. Tenderness should make one think of a scaphoid fracture. Not only is this the most common carpal bone injury, but the poor blood supply puts the bone at risk for avascular necrosis when injured. This fracture is commonly missed on x-ray, so this is an important physical finding to support further or repeated studies.

A 58-year-old gardener comes to your office for evaluation of a new lesion on her upper chest. The lesion appears to be "stuck on" and is oval, brown, and slightly elevated with a flat surface. It has a rough, wartlike texture on palpation. Based on this description, what is your most likely diagnosis? A) Actinic keratosis B) Seborrheic keratosis C) Basal cell carcinoma D) Squamous cell carcinoma

B) Seborrheic keratosis This is a typical description for seborrheic keratosis. The "stuck on" appearance and the rough, wartlike texture are key features for the diagnosis. They often produce a greasy scale when scratched with a fingernail, which further helps to distinguish them from other lesions. Frequently, these benign lesions actually meet several of the ABCDEs of melanoma, so it is important to distinguish these lesions to prevent unnecessary biopsy. It is important to consider biopsy whenever there is any doubt, though.

You are on rounds at the nursing home when you are asked to see an 83-year-old retired farmer who is bed-ridden. The aide is concerned because she has noticed an area on his coccyx that is broken down. You examine the skin and determine that there is a partial thickness skin loss, which involves the epidermis. What stage is this pressure ulcer? (A) Stage I (B) Stage II (C) Stage III (D) Stage IV

B) Stage II This patient has a stage II ulcer, which is a partial-thickness skin loss or ulceration that involves the epidermis, dermis, or both layers. A stage I ulcer is defined as intact skin that has a change in temperature, consistency, sensation, or color. The temperature can be warm or cool; the consistency can be firm or boggy; the patient may experience pain or itching; the color may be red, blue, or purple. A stage III ulcer is a fullthickness skin loss, with evidence of damage to or necrosis of subcutaneous tissue, which may extend to, but not through, the underlying muscle. A stage IV ulcer involves full-thickness skin loss with destruction, tissue necrosis, or damage to underlying muscle, bone, or supporting structures

A 32-year-old warehouse worker presents for evaluation of low back pain. He notes a sudden onset of pain after lifting a set of boxes that were heavier than usual. He also states that he has numbness and tingling in the left leg. He wants to know if he needs to be off of work. What test should you perform to assess for a herniated disc? A) Leg-length test B) Straight-leg raise C) Tinel's test D) Phalen's test

B) Straight-leg raise The straight-leg raise involves having the patient lie supine with the examiner raising the leg. If the patient experiences a sharp pain radiating from the back down the leg in an L5 or S1 distribution, that suggests the presence of a herniated disc

Mrs. Fletcher comes to your office with unilateral pain during chewing, which is chronic. She does not have facial tenderness or tenderness of the scalp. Which of the following is the most likely cause of her pain? A) Trigeminal neuralgia B) Temporomandibular joint syndrome C) Temporal arteritis D) Tumor of the mandible

B) Temporomandibular joint syndrome Temporomandibular joint syndrome is a very common cause of pain with chewing. Ischemic pain with chewing, or jaw claudication, can occur with temporal arteritis, but the lack of tenderness of the scalp overlying the artery makes this less likely. Trigeminal neuralgia can be associated with extreme tenderness over the branches of the trigeminal nerve. While a tumor of the mandible is possible, is it much less likely than the other choices.

Mrs. Anderson presents with an itchy rash which is raised and appears and disappears in various locations. Each lesion lasts for many minutes. What most likely accounts for this rash? A) Insect bites B) Urticaria, or hives C) Psoriasis D) Purpura

B) Urticaria, or hives This is a typical case of urticaria. The most unusual aspect of this condition is that the lesions "move" from place to place. This would be distinctly unusual for the other causes listed.

In assessing for an S4 heart sound with a stethoscope, the nurse would listen with the: A) bell at the base with the patient leaning forward B) bell at the apex with the patient in the L lateral position C) diaphragm in the aortic area with the patient sitting D) diaphragm in the pulmonic area with the patient supine

B) bell at the apex with the patient in the L lateral position The S4 is a ventricular filling sound. It occurs when atria contract late in diastole. It is heard immediately before S1. This is a very soft sound with a very low pitch. The nurse needs a good bell and must listen for it. It is heard best at the apex, with the person in the L lateral position.

A 62-year-old construction worker presents to your clinic, complaining of almost a year of chronic cough and occasional shortness of breath. Although he has had worsening of symptoms occasionally with a cold, his symptoms have stayed about the same. The cough has occasional mucous drainage but never any blood. He denies any chest pain. He has had no weight gain, weight loss, fever, or night sweats. His past medical history is significant for high blood pressure and arthritis. He has smoked two packs a day for the past 45 years. He drinks occasionally but denies any illegal drug use. He is married and has two children. He denies any foreign travel. His father died of a heart attack and his mother died of Alzheimer's disease. On examination you see a man looking slightly older than his stated age. His blood pressure is 130/80 and his pulse is 88. He is breathing comfortably with respirations of 12. His head, eyes, ears, nose, and throat examinations are unremarkable. His cardiac examination is normal. On examination of his chest, the diameter seems enlarged. Breath sounds are decreased throughout all lobes. Rhonchi are heard over all lung fields. There is no area of dullness and no increased or decreased fremitus. What thorax or lung disorder is most likely causing his symptoms? A) spontaneous pneumothorax B) chronic obstructive pulmonary disease (COPD) C) asthma D) pneumonia

B) chronic obstructive pulmonary disease (COPD) Chronic obstructive pulmonary disease (COPD)This disorder is insidious in onset and generally affects the older population with a smoking history. The diameter of the chest is often enlarged like a barrel. Percussing the chest elicits hyperresonance, and during auscultation there are often distant breath sounds. Coarse breath sounds of rhonchi are also often heard. It is important to quantify this patient's exercise capacity because it may affect his employment and also allows you to follow for progression of his disease. You must offer smoking cessation as an option.

A grandmother brings her 13-year-old grandson for evaluation. She noticed last week when he took off his shirt that his breastbone seemed collapsed. He seems embarrassed and says that it has been that way for awhile. He states he has no symptoms from it and that he just tries not to take off his shirt in front of anyone. He denies any shortness of breath, chest pain, or lightheadedness on exertion. His past medical history is unremarkable. He is in sixth grade and just moved in with his grandmother after his father was transferred for a work contract. His mother died several years ago in a car accident. He states that he does not smoke and has never touched alcohol. Examination shows a teenage boy appearing his stated age. Visual examination of his chest reveals that the lower portion of the sternum is depressed. Auscultation of the lungs and heart is unremarkable. What disorder of the thorax best describes these findings? A) barrel chest B) funnel chest (pectus excavatum) C) pigeon chest (pectus carinatum) D) thoracic kyphoscoliosis

B) funnel chest (pectus excavatum) Funnel chest is caused by a depression in the lower portion of the sternum. If severe enough there can be compression of the heart and great vessels, leading to murmurs on auscultation. This is usually only a cosmetic problem, but corrective surgeries can be performed if necessary.

A patient with a recent diagnosis of hypothyroidism presents to the clinic. During the skin assessment, you may expect the following skin finding: A) oily skin on face and trunk B) generalized areas of dry skin C) generalized areas of warmth over the skin D) skin that is a velvety texture

B) generalized areas of dry skin

You are wanting to assess cranial nerve VIII. How would you test this? A) lateral deviation of the eyes B) hearing C) facial movements D) test tasting

B) hearing CN 8 is the acoustic nerve, so testing hearing is the best way to test.

Which of the following conditions would produce a unilateral hyperresonant percussion note? A) lobar pneumonia B) large pneumothorax C) empyema D) COPD

B) large pneumothorax

A patient with longstanding COPD was told by another practitioner that his liver was enlarged and this needed to be assessed. Which of the following would be a true statement about the liver in COPD. A) liver becomes hard and therefore tympanic to percussion B) liver moves lower due to the hyperinflated lungs C) COPD causes liver cancer D) none of the above

B) liver moves lower due to the hyperinflated lungs In this patient, measuring the span of the liver saved the patient an involved workup, because it was normal. His history of COPD is consistent with flattening of the diaphragms, which pushed the liver edge down while the actual size of the liver remained the same. Percussing the lower border of the liver alone caused this referral, because it was assumed that the liver was enlarged.

Jane is an NP who observes a raised rash on the backs of her patient's knees covering an area greater than 1 cm. She charts these solid bumps as: A) macules B) papules C) plaques D) vesicles

B) papules

Joy is a nurse practitioner in a busy family practice clinic. He is charting on his patient, who appears to have a skin rash on her face. He charts the following: no fevers, no new lotions or soaps, no new medications, no recent sun exposure, no itching. His documentation is an example of charting: A) too much detail about the chief complaint B) pertinent negatives C) in a succinct writing style D) objective data

B) pertinent negatives

Which would be an example of secondary amenorrhea? A) never had a period B) pregnancy C) heavy menstrual bleeding C) intermenstrual bleeding

B) pregnancy Pregnancy is the number one cause of secondary amenorrhea (absence of periods after initially starting periods).

The following information is recorded in the health history: "The patient has had abdominal pain for 1 week. The pain lasts for 30 minutes at a time; it comes and goes. The severity is 7 to 8 on a scale of 1 to 10. It is accompanied by nausea and vomiting. It is located in the mid-epigastric area."Which of these categories does it belong to? A. Chief complaint B. Present illness C. Personal and social history D. Review of systems

B. Present illness This information describes the problem of abdominal pain, which is the present illness. The interviewer has obtained the location, timing, severity, and associated manifestations of the pain. The interviewer will still need to obtain information concerning the quality of the pain, the setting in which it occurred, and the factors that aggravate and alleviate the pain. You will notice that it does include portions of the pertinent review of systems, but because it relates directly to the complaint, it is included in the history of present illness.

For which of the following patients would a comprehensive health history be appropriate? A) A new patient with the chief complaint of "I sprained my ankle" B) An established patient with the chief complaint of "I have an upper respiratory infection." C) A new patient with the chief complaint of "I am here to establish care" D) A new patient with the chief complaint of "I cut my hand"

C) A new patient with the chief complaint of "I am here to establish care"

Phil comes to your office with left "shoulder pain." You find that the pain is markedly worse when his left arm is drawn across his chest (adduction). Which of the following would you suspect? A) Rotator cuff tear B) Subacromial bursitis C) Acromioclavicular joint involvement D) Adhesive capsulitis

C) Acromioclavicular joint involvement Adduction of the patient's arm across his chest can cause pain if the acromioclavicular joint is involved. In adhesive capsulitis, this maneuver may not be possible due to limited range of motion. Subacromial bursitis would present with tenderness inferior to the acromion. Rotator cuff injury would ordinarily not be associated with pain during this maneuver.

A 68-year-old retired truck driver comes to your office for evaluation of swelling in his legs. He is a smoker and has been taking medications to control his hypertension for the past 25 years. You are concerned about his risk for peripheral vascular disease. Which of the following tests are appropriate to order to initially evaluate for this condition? A) Venogram B) CT scan of the lower legs C) Ankle-brachial index (ABI) D) PET scan

C) Ankle-brachial index (ABI) The ankle-brachial index (ABI) is a good test for obtaining information about significant stenosis in the vessels of the lower extremities. Sixteen percent of patients with known peripheral vascular disease also have coronary artery disease.

Ms. Whiting is a 68 year old who comes in for her usual follow-up visit. You notice a few flat red and purple lesions, about 6 centimeters in diameter, on the ulnar aspect of her forearms but nowhere else. She doesn't mention them. They are tender when you examine them. What should you do? A) Conclude that these are lesions she has had for a long time. B) Wait for her to mention them before asking further questions. C) Ask how she acquired them. D) Conduct the visit as usual for the patient.

C) Ask how she acquired them. These are consistent with ecchymoses, or bruises. It is important to ask about antiplatelet medications such as aspirin, trauma history, and history of blood disorders in the patient and her family. Because of the different ages of the bruises and the isolation of them to the ulnar forearms, these may be a result of abuse or other violence. It is your duty to investigate the cause of these lesions.

You are assessing a 59-year-old gas station owner for atherosclerosis in the lower extremities. In which of the following locations would the patient's pain make you concerned for this disease process? A) Thigh B) Knee C) Calf D) Ankle

C) Calf Pain in the calf is the most common site for claudication; however, there could be pain in the buttock, hip, thigh, or foot, depending on the level of the obstruction. The absence of this pain does not rule out significant vascular disease, and actually the minority of these patients are symptomatic.

Marion presents to your office with back pain associated with constipation and urinary retention. Which of the following is most likely? A) Sciatica B) Epidural abscess C) Cauda equina D) Idiopathic low back pain

C) Cauda equina The presence of bowel and bladder symptoms associated with back pain is worrisome and should suggest impingement of nerve roots S2-S4. For this reason idiopathic low back pain is unlikely. Epidural abscess may present with midline pain which can be increased with percussion over the spinous processes. Sciatica is associated with pain which radiates into the buttocks and/or down the posterior leg in the S1 distribution.

A 50-year-old realtor comes to your office for evaluation of neck pain. She was in a motor vehicle collision 2 days ago and was assessed by the emergency medical technicians on site, but she didn't think that she needed to go to the emergency room at that time. Now, she has severe pain and stiffness in her neck. On physical examination, you note pain and spasm over the paraspinous muscles on the left side of the neck, and pain when you make the patient do active range of motion of the cervical spine. What is the most likely cause of this neck pain? A) Simple stiff neck B) Aching neck C) Cervical sprain D) Cervical herniated disc

C) Cervical sprain The patient most likely has an acute whiplash injury secondary to the collision. The features of the physical examination, local tenderness and pain on movement, are consistent with cervical sprain.

You are obtaining an arterial blood gas in the radial artery on a retired cab driver who has been hospitalized in the intensive care unit for a stroke. You are concerned about the possibility of arterial insufficiency. You perform the Allen test. This means that you: A) Checked for patency of the radial artery B) Checked for patency of the brachial artery C) Checked for patency of the ulnar artery D) Checked for patency of the femoral artery

C) Checked for patency of the ulnar artery The Allen test is for determining patency of the ulnar artery before puncturing the radial artery. In the event of an occlusion in the radial artery system, the ulnar artery can provide adequate blood flow.

Mr. Larson is a 42-year-old widowed father of two children, ages 4 and 11. He works in a sales office to support his family. Recently he has injured his back and you are thinking he would benefit from physical therapy, three times a week, for an hour per session. What wouldbe your next step? A) Write the physical therapy prescription. B) Have your office staff explain directions to the physical therapy center. C) Discuss the plan with Mr. Larson. D) Tell Mr. Larson that he will be going to physical therapy three times a week.

C) Discuss the plan with Mr. Larson.

A 28-year-old graduate student comes to your clinic for evaluation of pain "all over." With further questioning, she is able to relate that the pain is worse in the neck, shoulders, hands, low back, and knees. She denies swelling in her joints; she states that the pain is worse in the morning; there is no limitation in her range of motion. On physical examination, she has several points on the muscles of the neck, shoulders, and back that are tender to palpation; muscle strength and range of motion are normal. Which of the following is likely the cause of her pain? A) Rheumatoid arthritis B) Osteoarthritis C) Fibromyalgia D) Polymyalgia rheumatica

C) Fibromyalgia The patient has pain in specific trigger point areas on the muscles, with normal strength and range of motion. This is an indication for fibromyalgia.

A 12-year-old presents to the clinic with his father for evaluation of a painful lump in the left eye. It started this morning. He denies any trauma or injury. There is no visual disturbance. Upon physical examination, there is a red raised area at the margin of the eyelid that is tender to palpation; no tearing occurs with palpation of the lesion. Based on this description, what is the most likely diagnosis? A) Dacryocystitis B) Chalazion C) Hordeolum D) Xanthelasma

C) Hordeolum A hordeolum, or stye, is a painful, tender, erythematous infection in a gland at the margin of the eyelid.

Two weeks ago, Mary started a job which requires carrying 40-pound buckets. She presents with elbow pain worse on the right. On examination, it hurts her elbows to dorsiflex her hands against resistance when her palms face the floor. What condition does she have? A) Medial epicondylitis (golfer's elbow) B) Olecranon bursitis C) Lateral epicondylitis (tennis elbow) D) Supracondylar fracture

C) Lateral epicondylitis (tennis elbow) Mary's injury probably occurred by lifting heavy buckets with her palms down (toward the bucket). This caused her chronic overuse injury at the lateral epicondyle. Medial epicondylitis has reproducible pain when palmar flexion against resistance is performed and also features tenderness over the involved epicondyle. Olecranon bursitis produces erythema and swelling over the olecranon process. A supracondylar fracture of the humerus is a major injury and would present more acutely

You are examining an unconscious patient from another region and notice Beau's lines, a transverse groove across all of her nails, about 1 cm from the proximal nail fold. What would you do next? A) Conclude this is caused by a cultural practice. B) Conclude this finding is most likely secondary to trauma. C) Look for information from family and records regarding any problems which occurred 3 months ago. D) Ask about dietary intake.

C) Look for information from family and records regarding any problems which occurred 3 months ago. These lines can provide valuable information about previous significant illnesses, some of which are forgotten or are not able to be reported by the patient. Because the fingernails grow at about 0.1 mm per day, you would ask about an illness 100 days ago. This patient may have been hospitalized for endocarditis or may have had another significant illness which should be sought. Trauma to all 10 nails in the same location is unlikely. Dietary intake at this time would not be related to this finding. Do not assume a finding is necessarily related to a patient's culture unless you have good knowledge of that culture.

You are assessing a patient with diffuse joint pains and want to make sure that only the joints are the problem, and that the pain is not related to other diseases. Which of the following is a systemic cause of joint pain? A) Gout B) Osteoarthritis C) Lupus D) Spondylosis

C) Lupus Lupus is a systemic disease, one symptom of which may be joint pain. It is important to consider the presence of a systemic illness when a patient presents with arthritis

A 33-year-old construction worker comes for evaluation and treatment of acute onset of low back pain. He notes that the pain is an aching located in the lumbosacral area. It has been present intermittently for several years; there is no known trauma or injury. He points to the left lower back. The pain does not radiate and there is no numbness or tingling in the legs or incontinence. He was moving furniture for a friend over the weekend. On physical examination, you note muscle spasm, with normal deep tendon reflexes and muscle strength. What is the most likely cause of this patient's low back pain? A) Herniated disc B) Compression fracture C) Mechanical low back pain D) Ankylosing spondylitis

C) Mechanical low back pain The case is an example of mechanical low back pain; in a large percentage of cases there is no known underlying cause. The pain is often precipitated by moving, lifting, or twisting motions and relieved by rest.

A young man comes to you with an extremely pruritic rash over his knees and elbows which has come and gone for several years. It seems to be worse in the winter and improves with some sun exposure. On examination, you notice scabbing and crusting with some silvery scale, and you are observant enough to notice small "pits" in his nails. What would account for these findings? A) Eczema B) Pityriasis rosea C) Psoriasis D) Tinea infection

C) Psoriasis This is a classic presentation of plaque psoriasis. Eczema is usually over the flexor surfaces and does not scale, whereas psoriasis affects the extensor surfaces. Pityriasis usually is limited to the trunk and proximal extremities. Tinea has a much finer scale associated with it, almost like powder, and is found in dark and moist areas.

A 72-year-old teacher comes to a skilled nursing facility for rehabilitation after being in the hospital for 6 weeks. She was treated for sepsis and respiratory failure and had to be on the ventilator for 3 weeks. You are completing your initial assessment and are evaluating her skin condition. On her sacrum there is full-thickness skin loss that is 5 cm in diameter, with damage to the subcutaneous tissue. The underlying muscle is not affected. You diagnose this as a pressure ulcer. What is the stage of this ulcer? A) Stage 1 B) Stage 2 C) Stage 3 D) Stage 4

C) Stage 3 A stage 3 ulcer is a full-thickness skin loss with damage to or necrosis of subcutaneous tissue that may extend to, but not through, the underlying muscle.

Which of the following synovial joints would be an example of a condylar joint? A) Hip B) Interphalangeal joints of the hand C) Temporomandibular joint D) Intervertebral joint

C) Temporomandibular joint The TMJ is an example of a condylar joint because it involves the movement of two surfaces which are not dissociable. The hip would be an example of a spheroidal joint and the interphalangeal joints of the hand are hinge joints. The intervertebral joints are not synovial joints at all, but rather cartilaginous joints.

Sarah presents with left lateral knee pain and has some locking in full extension. There is tenderness over the medial joint line. When the knee is extended with the foot externally rotated and some valgus stress is applied, a click is noted. What is the most likely diagnosis? A) Torn anterior cruciate ligament B) Torn posterior cruciate ligament C) Torn medial meniscus D) Torn lateral meniscus

C) Torn medial meniscus This maneuver is called the McMurray test. Along with the medial joint line tenderness, you should suspect a medial meniscus injury. Cruciate ligament tears should cause an anterior or posterior "drawer sign." Although we can't rule out a lateral meniscus tear, the tenderness along the medial joint line makes this the more likely site of injury.

Coordination of muscle movement requires that four areas of the nervous system function in an integrated way. Coordinating eye, head, and body movements applies to which area of the nervous system? A) Motor system B) Cerebellar system C) Vestibular system D) Sensory system

C) Vestibular system: balance and coordinating eye, head, and body movements Motor system: muscle strength Cerebellar system: rhythmic movement and steady posture Sensory system: position sense

The examiner inspects the tongue for symmetry. Asymmetry of the tongue indicates a lesion on which cranial nerve? A) X B) XI C) XII D) IX

C) XII

During an assessment of a 68-year-old man with a recent onset of right-sided weakness, the nurse hears a blowing, swishing sound with the bell of the stethoscope over the left carotid artery. This finding would indicate: A) fluid volume overload B) ventricular hypertrophy C) blood flow turbulence D) valvular disorder

C) blood flow turbulence A bruit is a blowing, swishing sound indicating blood flow turbulence; normally none is present.

Which lung sound possesses the characteristics of being louder and higher in pitch, with a short silence between inspiration and expiration and with expiration being longer than inspiration? A) bronchovesicular B) vesicular C) bronchial D) tracheal

C) bronchial

A newborn male infant is examined and found not to have testicles present in the scrotal sac. What should the nurse do next? A) call urologist for a stat surgery B) perform a digital rectal exam C) check the inguinal area and see if she can milk down the testicles D) give up, it's not important

C) check the inguinal area and see if she can milk down the testicles

A light is pointed at a patient's pupil, which contracts. It is also noted that the other pupil contracts though it is not directly exposed to bright light. What is this phenomenon? A) direct reaction B) near reaction C) consensual reaction D) accommodation

C) consensual reaction

The changes normally associated with menopause occur generally because the decreased level of: A) insulin. B) fibroids. C) estrogen. D) growth hormone.

C) estrogen.

The mother of a 3-month-old infant states that her baby has not been gaining weight. With further questioning, the nurse finds that the infant falls asleep after nursing and wakes up after a short amount of time, hungry again. What other information would the nurse want to have? A) infant's sleeping position B) amount of background noise when eating C) presence of dyspnea or diaphoresis when sucking D) sibling history of eating disorders

C) presence of dyspnea or diaphoresis when sucking To screen for heart disease in an infant, focus on feeding. Note fatigue during feeding. An infant with heart failure take fewer ounces each feeding, becomes dyspneic with sucking, may be diaphoretic and then falls into exhausted sleep and awakens after a short time hungry again.

The area of the nervous system that is responsible for mediating reflexes is the: A) medulla B) cerebellum C) spinal cord D) cerebral cortex

C) spinal cord The spinal cord is the main highway for ascending and descending fiber tracts that connect the brain to the spinal nerves, and it mediates reflexes.

During the interview with a female patient, the nurse gathers data that indicate that the patient is perimenopausal. Which of these statements made by this patient leads to this conclusion? A)"I have noticed that my muscles ache at night when I go to bed." B)"I will be very happy when I can stop worrying about having a period." C)"I have been noticing that I sweat a lot more than I used to, especially at night." D)"I have only been pregnant twice, but both times I had breast tenderness as my first symptom." ANS: CHormone shifts occur during the perimenopausal period, and associated symptoms of menopause may occur, such as hot flashes, night sweats, numbness and tingling, headache, palpitations, drenching sweats, mood swings, vaginal dryness, and itching. The other responses are not correct.

C)"I have been noticing that I sweat a lot more than I used to, especially at night." Hormone shifts occur during the perimenopausal period, and associated symptoms of menopause may occur, such as hot flashes, night sweats, numbness and tingling, headache, palpitations, drenching sweats, mood swings, vaginal dryness, and itching. The other responses are not correct.

An obese 55-year-old woman went through menarche at age 16 and menopause 2 years ago. She is concerned because an aunt had severe osteoporosis. Which of the following is a risk factor for osteoporosis? A) Obesity B) Late menopause C) Having an aunt with osteoporosis D) Delayed menarche

D) Delayed menarche Obesity and late menopause are not associated with osteoporosis. Having a first-degree relative with osteoporosis is a risk factor, but an aunt is a second-degree relative. Delayed menarche is the only choice which is a known risk factor for osteoporosis

You are beginning the examination of the skin on a 25-year-old teacher. You have previously elicited that she came to the office for evaluation of fatigue, weight gain, and hair loss. You strongly suspect that she has hypothyroidism. What is the expected moisture and texture of the skin of a patient with hypothyroidism? A) Moist and smooth B) Moist and rough C) Dry and smooth D) Dry and rough

D) Dry and rough A patient with hypothyroidism is expected to have skin that is dry as well as rough. This is a good example of how the skin can give clues to systemic diseases.

A patient complains of knee pain on your arrival in the room. What should your first sentence be after greeting the patient? A) How much pain are you having? B) Have you injured this knee in the past? C) When did this first occur? D) Could you please describe what happened?

D) Could you please describe what happened? When looking into a complaint, it is best to start with an invitation for the patient to tell you in his or her own words. More specific questions should be used later in the interview to fill in any gaps.

A 48-year-old grocery store manager comes to the clinic complaining of her head being "stuck" to one side. She says that today she was doing her normal routine when it suddenly felt like her head was being moved to her left and then it just stuck that way. She says it is somewhat painful because she cannot move it back to a normal position. She denies any recent neck trauma. Her past medical history consists of type 2 diabetes and gastroparesis (slow-moving peristalsis in the digestive tract, seen in diabetes). She is taking oral medication for each. She is married with three children. She denies tobacco, alcohol, or drug use. Her father has diabetes and her mother passed away from breast cancer. Her children are healthy. Examination reveals a slightly overweight Hispanic woman appearing her stated age. Her head is twisted grotesquely to her left; otherwise, her examination is normal. What form of involuntary movement does she have? A) Chorea B) Asbestosis C) Tic D) Dystonia

D) Dystonia Dystonic movements are similar to athetoid movement, but often involve larger portions of the body, including the trunk. Grosteque twisted posture may result.

You are working in a college health clinic and seeing a young woman with a red, painful, swollen DIP joint on the left index finger. There are also a few papules, pustules, and vesicles on reddened bases, located on the distal extremities. This would be consistent with which of the following? A) Lyme disease B) Systemic lupus erythematosus C) Hives (urticaria) D) Gonococcal arthritis

D) Gonococcal arthritis The presentation of a monoarthritis in this age group should lead one to think of gonococcal disease. Skin findings are often seen in conjunction with arthritis. Lyme disease is associated with an expanding erythematous patch. Lupus is associated with a "butterfly" rash on the cheeks, while serum sickness and drug reactions can be associated with hives.

A 77-year-old retired nurse has an ulcer on a lower extremity that you are asked to evaluate when you do your weekly rounds at a local long-term care facility. All of the following are responsible for causing ulcers in the lower extremities except for which condition? A) Arterial insufficiency B) Venous insufficiency C) Diminished sensation in pressure points D) Hypertension

D) Hypertension Hypertension is not directly associated with the formation of ulcers. It is an indirect risk factor if it is uncontrolled for a long time and associated with atherosclerosis, because it can lead to arterial insufficiency or neuropathy.

A 19-year-old college sophomore comes to the clinic for evaluation of joint pains. The student has been back from spring break for 2 weeks; during her holiday, she went camping. She notes that she had a red spot, shaped like a target, but then it started spreading, and then the joint pains started. She used insect repellant but was in an area known to have ticks. She has never been sick and takes no medications routinely; she has never been sexually active. What is the most likely cause of her joint pain? A) Trauma B) Gonococcal arthritis C) Psoriatic arthritis D) Lyme disease

D) Lyme disease Lyme disease is characterized by a target-shaped red spot at the site of the bite, which disappears, then reappears and starts spreading (erythema migrans). Lyme disease can also result in joint pain as well as cardiac and neurologic manifestations

A patient who presents to clinic complaining of hand pain says she was told by a friend that it is most likely carpal tunnel syndrome. Upon assessing the patient, you note the following findings. Which would be suggestive of carpal tunnel syndrome? A) Hand pain when holding both hands in acute extension B) Numbness and tingling when tapping over the course of the radial nerve C) Symptoms related to compression are evident in all of the fingers D) None of the above

D) None of the above The symptoms of carpal tunnel syndrome are: -Numbness or tingling with pressing backs of hands together in acute flexion for 60 seconds -Tingling with tapping over the median nerve as it enters the carpal tunnel -Pain or numbness of the first three fingers of the hand, but not in the palm

During an interview, the nurse states, "You mentioned shortness of breath. Tell me more about that." Which verbal skill is used with this statement? A) Reflection B) Facilitation C) Direct question D) Open-ended question

D) Open-ended question The open-ended question asks for narrative information. It states the topic to be discussed but only in general terms. The nurse should use it to begin the interview, to introduce a new section of questions, and whenever the person introduces a new topic

A 58-year-old man comes to your office complaining of bilateral back pain that now awakens him at night. This has been steadily increasing for the past 2 months. Which one of the following is the most reassuring in this patient with back pain? A) Age over 50 B) Pain at night C) Pain lasting more than 1 month or not responding to therapy D) Pain that is bilateral

D) Pain that is bilateral While bilateral pain can be associated with serious illness, it is not one of the "red flags" of back pain. Red flags should make one suspicious for serious underlying systemic disease such as cancer, infection, or others. This list includes: age over 50, history of cancer, unexplained weight loss, pain lasting more than 1 month or not responding to treatment, pain at night or increased by rest, history of intravenous drug use, or presence of infection. The presence of one of these with low back pain indicates a 10% probability of a serious systemic disease.

A 55-year-old secretary with a recent history of breast cancer, for which she underwent surgery and radiation therapy, and a history of hypertension comes to your office for a routine checkup. Which of the following aspects of the physical are important to note when assessing the patient for peripheral vascular disease in the arms? A) Femoral pulse, popliteal pulse B) Dorsalis pedis pulse, posterior tibial pulse C) Carotid pulse D) Radial pulse, brachial pulse

D) Radial pulse, brachial pulse This is an important aspect of physical examination to assess for peripheral vascular disease. This patient is at risk for disease in this distribution because of her recent radiation therapy.

A 15-year-old high school sophomore presents to the emergency room with his mother for evaluation of an area of blood in the left eye. He denies trauma or injury but has been coughing forcefully with a recent cold. He denies visual disturbances, eye pain, or discharge from the eye. On physical examination, the pupils are equal, round, and reactive to light, with a visual acuity of 20/20 in each eye and 20/20 bilaterally. There is a homogeneous, sharply demarcated area at the lateral aspect of the base of the left eye. The cornea is clear. Based on this description, what is the most likely diagnosis? A) Conjunctivitis B) Acute iritis C) Corneal abrasion D) Subconjunctival hemorrhage

D) Subconjunctival hemorrhage A subconjunctival hemorrhage is a leakage of blood outside of the vessels, which produces a homogenous, sharply demarcated bright red area; it fades over several days, turning yellow, then disappears. There is no associated eye pain, ocular discharge, or changes in visual acuity; the cornea is clear. Many times it is associated with severe cough, choking, or vomiting, which increase venous pressure. It is rarely caused by a serious condition, so reassurance is usually the only treatment necessary.

A high school soccer player "blew out his knee" when the opposing goalie's head and shoulder struck his flexed knee while the goalie was diving for the ball. All of the following structures were involved in some way in his injury, but which of the following is actually an extra-articular structure? A) Synovium B) Joint capsule C) Juxta-articular bone D) Tendons

D) Tendons Extra-articular structures include the periarticular ligaments, tendons, bursae, muscle, fascia, bone, nerve, and overlying skin. The articular structures include the joint capsule and articular cartilage, the synovium and synovial fluid, intra-articular ligaments, and juxta-articular bone.

You are performing a routine check-up on an 81-year-old retired cotton farmer in the vascular surgery clinic. You note that he has a history of chronic arterial insufficiency. Which of the following physical examination findings in the lower extremities would be expected with this disease? A) Normal pulsation B) Normal temperature C) Marked edema D) Thin, shiny, atrophic skin

D) Thin, shiny, atrophic skin Thin, shiny, atrophic skin is more commonly seen in chronic arterial insufficiency; in chronic venous insufficiency the skin often has a brown pigmentation and may be thickened.

A 50-year-old woman presents with "left hip pain" of several weeks duration. There is marked tenderness when you press over her proximal lateral thigh. What do you think she has? A) Osteoarthritis B) Rheumatoid arthritis C) Sciatica D) Trochanteric bursitis

D) Trochanteric bursitis Bursitis is usually accompanied by tenderness on examination. This location is consistent with trochanteric bursitis. Osteoarthritis would generally not be tender and would more likely have decreased range of motion. Rheumatoid arthritis and sciatica would not likely be tender over this area.

An elevated PSA can be caused by: A) ejaculation B) BPH C) prostatitis D) any of the above

D) any of the above

When listening to heart sounds, the nurse knows that S1: A) is louder than S2 at the base of the heart. B) indicates the beginning of diastole. C) is caused by closure of the semilunar valves. D) coincides with the carotid artery pulse.

D) coincides with the carotid artery pulse. S1 coincides with the carotid artery pulse. S1 is the start of systole and is louder than S2 at the apex of the heart; S2 is louder than S1 at the base. The nurse should feel the carotid artery pulse gently while auscultating at the apex; the sound heard as each pulse is felt is S1.

In assessing a 70-year-old, the nurse finds the following: blood pressure 140/100 mm Hg; heart rate 104 and slightly irregular; split S2. Which of these findings can be explained by expected hemodynamic changes related to age? A) increase in diastolic blood pressure B) increase in resting heart rate C) decrease in diastolic blood pressure D) increase in systolic blood pressure

D) increase in systolic blood pressure systolic/diastolic With aging, there is an increase in systolic blood pressure. No significant change in diastolic pressure or resting heart rate occurs with age. Cardiac output at rest is not changed with aging.

A 60-year-old man has just been told he has benign prostatic hypertrophy (BPH). He has a friend who just died from cancer of the prostate, and he is concerned this will happen to him. How should the nurse respond? A)"The swelling in your prostate is only temporary and will go away." B)"We will treat you with chemotherapy so we can control the cancer." C)"It would be very unusual for a man your age to have cancer of the prostate." D)"The enlargement of your prostate is caused by hormone changes and not cancer."

D)"The enlargement of your prostate is caused by hormone changes and not cancer."

A mother of a 1-month-old infant asks the nurse why it takes so long for infants to learn to roll over. The nurse knows that the reason for this is: a.A demyelinating process must be occurring with her infant. b.Myelin is needed to conduct the impulses, and the neurons of a newborn are not yet myelinated. c.The cerebral cortex is not fully developed; therefore, control over motor function gradually occurs. d.The spinal cord is controlling the movement because the cerebellum is not yet fully developed.

b.Myelin is needed to conduct the impulses, and the neurons of a newborn are not yet myelinated.

The _____, among other methods, has been extensively validated for use in identifying alcoholism a. clinical formulation b. FACE risk profile c. CAGE questionnaire d. mental disorder

c. CAGE questionnaire

A 30-year-old woman tells the nurse that she has been very unsteady and has had difficulty in maintaining her balance. Which area of the brain that is related to these findings would concern the nurse? a.Thalamus b.Brainstem c.Cerebellum d.Extrapyramidal tract

c.Cerebellum The cerebellar system coordinates movement, maintains equilibrium, and helps maintain positive posture.


संबंधित स्टडी सेट्स

Market Motive Mobile Marketing Practitioner

View Set

Technical Support Fundamentals: Week 1. How to Count in Binary

View Set

Ch 12: Drug Therapy: Immunizations

View Set

Communication PrepU (w/o explanations)

View Set

CSIT 216 Ch 1 - Introduction to Computers and Programming

View Set